Download as pdf or txt
Download as pdf or txt
You are on page 1of 38

LSAT

PrepTest 15
Explained
a guide to the june, 1995 exam
© 2006 Kaplan, Inc.

All rights reserved. No part of this book may be reproduced in any form, by photostat, microfilm, xerography, or any other
means, or incorporated into any information retrieval system, electronic or mechanical, without the written permission of
Kaplan, Inc. LSAT is a registered trademark of the Law School Admission Council.
Section I: Reading Comprehension

SECTION I: • Actually, the only examinees who screwed up


were those who saw that the topic was science
READING COMPREHENSION and, knee-jerk, hightailed it to another passage. A
word to the wise: Even science passages can be
Passage 1: What Happened manageable.
to the Dinosaurs? • Whenever the author describes several different
points of view, take care to ascertain her views
Questions 1–7 on them: Which, if any, does she especially favor?
Here, it’s not too tough to detect the author’s
Topic and Scope: Dinosaur extinction; specifically, enthusiasm for the volcano theory, but other
various different contemporary theories for dinosaurs’ passages may be more challenging in this regard.
disappearance.
The Questions:
Purpose and Main Idea: The author spends most of the
time explaining and touting the newest of three 1. (C)
theories (the “volcanic-eruption theory”) as to why the
This choice is a nice paraphrase of information in
dinosaurs died out. Two earlier theories are described
paragraph 2, which describes the nuts and bolts of the
as well, but the author seems to have the most
“meteorite-impact theory.”
confidence in this new one.
(A) Au contraire. Lines 5–9 demonstrate that the
Paragraph Structure: Each of the first three “climactic theor y”—a theor y that preceded the
paragraphs is devoted to a thumbnail description of a “meteorite-impact theor y”—could account for the
theory: the “climatic theory” (paragraph 1) that held extinction of ocean species at the end of the
sway until the 1980s; the “meteorite-impact theory” Cretaceous era.
(paragraph 2) popular in the ’80s; and the “volcanic- (B) refers to the details of the “volcanic-eruption
eruption theory” (paragraphs 3–4), which is supported theory,” a new theory intended to take the place of the
by newly-discovered evidence in India. The introductory older “meteorite-impact theory” of the 1980s.
keyword “Moreover” leads us to expect more
(D) Au contraire aussi. Paragraph 1 says that
information about the new theory, and that’s just what
researchers initially thought that the drop in sea level
we get: We learn that the evidence for the other two
thought to be behind the mass extinction episode at
theories (the change in sea level and the iridium
the end of the Cretaceous era was the result of
deposits) supports, or at least is consistent with, the
“noncatastrophic geological processes.”
eruption theory as well.
(E) mistakenly attributes an element of the “volcanic-
The Big Picture: eruption theory” to the “meteorite-impact theory.”
• Sometimes the LSAT writers throw you a real
curve when a Reading Comp. section begins—a 2. (A)
really complex passage, or one that’s way beyond The depths of the Earth’s mantle are described in two
most everyone’s ken. Happily for the June, 1995 places, line 28 and line 40, and we hope you kept
test takers, that wasn’t the case here. The reading past the first reference because the right
passage’s structure is as straightforward as can answer is yielded by the second. Lines 40–41 are
be, what with the theories and keyword choice (A) almost word-for-word.
“Moreover” clearly signaling the key shifts; and (B) Only the element iridium is described here, and in
thanks to Jurassic Park, this is hardly a topic that fact that’s an element that is present in the mantle but
most would find either unfamiliar or scary. On rare on the Earth’s surface. (B) gets it all cockeyed.
your LSAT, the testmakers may not be so obliging. (C) According to lines 27–28, the mantle is rather
Just remember: At least one of those passages unstable, and we get no information as to whether
will be of “low difficulty,” like this one, meaning things are more stable closer to the surface.
that most people are expected to do well on it.
Find it, and attack it early in the section, even if (D) The only reference to CO2 comes in line 24, before
it’s not printed first. the mantle is even mentioned, and all we’re told is that
it’s released with lava. Yes, the lava eruption is
triggered by mantle instability, but that’s a long way
from what (D) is saying.

1
PrepTest 15 Explained

(E) As with (C), we’re given no comparison between 5. (D)


the mantle and the upper regions. And, anyhow, the This choice is an excellent paraphrase of the
mantle (when heated by the core) becomes less dense information contained in lines 42–47.
in places, not “uniformly” so.
(A) is beyond the scope of the passage, which only
discusses possible developments at the end of the
3. (D)
Cretaceous era.
Lines 39–43 say that the Earth’s mantle is rich in
(B) According to the “volcanic-eruption theory,” the
iridium, while its surface doesn’t have much of the
increase in atmospheric iridium and the drop in sea
stuff. Hence, we can infer that Cretaceous era lava
level were both caused by volcanic activity; there was
(lava is just heated rock from the Earth’s interior) “was
no direct connection between the drop in sea level and
richer in iridium” than surface rock.
the rise in atmospheric iridium levels.
(A) is beyond the scope of the passage, which never
(C) Again, the extinction of ocean species at the end
compares Cretaceous era lava with meteorites in
of the Cretaceous era, according to the volcanic theory,
terms of their respective carbon dioxide content.
was due to volcanic activity, not to rising levels of iridium
(B) Au contraire. Lines 29–32 indicate that lava is no in the atmosphere.
more dense than—and may well be less dense than—
(E) The “volcanic-eruption theory” never claims that
the molten rock just above the Earth’s core.
iridium is released into the atmosphere through “normal
(C) According to the “volcanic-eruption theory,” the geological processes.” This last phrase is associated
Earth’s climate was changed by massive volcanic with the “climactic theory” discussed in paragraph 1.
eruptions, not by the release of iridium hexaflouride,
which was simply a byproduct of those eruptions. 6. (B)
(E) The “volcanic-eruption theory” states that Cretaceous Lines 54–57 say that, according to the “volcanic-
era lava distributed iridium more evenly on the Earth’s eruption theory,” the gradual fall in sea level was the
surface than meteorite impacts could have; it doesn’t, result of the upward movement of diapirs toward the
however, claim that the lava was richer in iridium than Earth’s surface.
meteorites.
(A) Lines 26–34 demonstrate that this theory makes
assumptions about the temperature of molten rock just
4. (A)
above the Earth’s core.
What could be more of a slam dunk? If you pre-phrased
(C) is beyond the scope of the text, which doesn’t
an answer after reading the passage but before
discuss episodes of mass extinction before the one at
reading the choices, it’s possible that you came up
the end of the Cretaceous era.
with (A) almost word-for-word.
(D) While the “volcanic-eruption theory” explains the
(B), (C) “Attacking”? “Inadequacies”? The author isn’t
distribution of iridium on the Earth’s surface, it doesn’t
critical of any of the theories. She’s just reporting on
try to explain either the relative scarcity of this element
which theories have held sway when. And (B)’s
on the Earth’s surface or its relative abundance in
implication that more than one theory was popular
meteorites.
before the 1980s is just inaccurate, as line 10 makes
clear. (E) According to the “volcanic-eruption theory,” iridium
should be distributed relatively evenly in this layer of
(D) implies that the topic and scope of the passage
clay.
focus on general principles of scientific inquiry, and the
relationship between evidence and theories. But no
7. (B)
“general assertion” is made; this passage is, as noted
above, a straightforward examination of dinosaur The “volcanic-eruption theory” claims that volcanic
extinction theories, based largely on chronology. activity caused the climactic changes that resulted in
the mass extinction that occurred at the end of the
(E) is a mess. No “skepticism” is confirmed, and the
Cretaceous era. If similar volcanic activity had occurred
“view held prior to the 1980s” is but one of the three
at other times, but had not resulted in climactic
theories explained, not the highlight of the passage.
changes, then we’d have to think twice about whether
this theory is in fact on the right track.

2
Section I: Reading Comprehension

(A) The “volcanic-eruption theory” claims that iridium


hexaflouride comes from the Earth’s interior. Hence,
the theor y wouldn’t be called into question if
meteorites were found to have only minor quantities of
the stuff. If anything, this evidence would tend to
strengthen the theory.
(C) The existence of other episodes of mass extinction
would have no bearing on the “volcanic-eruption theory,”
which (so far as we know, anyway) concerns only the
mass extinction at the end of the Cretaceous era.
(D) The “volcanic-eruption theory” doesn’t deny that
meteorites hit the Earth at the end of the Cretaceous
era; it simply claims that, however frequent these impacts
may have been, they can’t account for the physical
evidence associated with the mass extinction at the
end of this era.
(E) If marine species are in fact more vulnerable to
sudden changes in sea level than to gradual changes,
this fact would in no way undermine the notion that
they could’ve succumbed to a gradual change at the
end of the Cretaceous era, as the “volcanic-eruption
theory” suggests.

3
PrepTest 15 Explained

Passage 2: Women and Folklore • Always remember: If a difficult phrase or concept


is important enough, the author will define it
Questions 8–15 more clearly for you before too long. (And if it’s
not important, who cares what it means?)
Topic and Scope: Folklore studies; specifically, a
recent shift in interest to women folklorists. The Questions:
Purpose and Main Idea: The author sets out to provide 8. (E)
evidence as to the increased interest in women
This choice nicely captures the author’s topic, scope,
folklorists. While the passage begins with a description
and purpose.
of another shift—to “the folk” (performers) and away
from just “the lore” (stories)—notice that the author (A) According to lines 7–9, folk performers, as opposed
never gets back to that until paragraph 4. The bulk of to just their works, began to get attention from the
his interest is women performers. early 1970s. The increased emphasis on women, the
text implies, is an even more recent phenomenon.
Paragraph Structure: Paragraph 1 lays out the topic, Besides, the precise date when scholars first began to
scope, and shifts described above, and paragraph 2 give attention to the contributions of women folklorists
provides specifics as to the shift to interest in women: is a mere detail.
a specific element of what’s been true in folklore “until (B) nicely sums up the gist of paragraph 4, but certainly
recently” (compare line 2 to line 13), and a specific doesn’t qualify as the passage’s main point.
example of how things are different.
(C) Lines 38–48 indicate that recent works about
Paragraph 3 first takes us back a century—to show women folklorists don’t focus primarily on the “the
that this interest in women folklorists isn’t unique to problems of repertoire analysis.”
the present day—and then highlights the content, pros,
(D) The text claims that folklore studies have shifted
and cons of two contemporary studies of two women
from an almost exclusive emphasis on folklore itself to
folklorists. Paragraph 4’s focus is defined by lines 52–53
an interest in the people who transmit it. (D) makes a
—brief speculation as to the potential “result[s] of this
quite different claim.
line of study”—both the increased emphasis on
women and the interest in “the folk” rather than just
9. (E)
“the lore.”
Paragraph 2’s purpose, as noted above, is to provide
The Big Picture: examples of the new trend toward focusing on women
• Early on you should listen keenly for keywords in folklore studies, and the author asserts that The
that hint at overall structure. Here, for instance, Dynamics of Folklore is a “telling” example—“perhaps
“It has become…a truism…until recently” clearly more telling,” even, than several recent studies in
tells you that, in the author’s mind, that truism no which women are central.
longer holds. Figure out the truism and you have (A) What’s “too soon to tell” is the extent of the shift
the author’s p.o.v. in the bag! (line 18–20), but the author has no such reservations
• You must listen for, recognize, and interpret as to whether women are coming to the forefront.
keywords throughout your reading, of course, not (B) Weigle & Farrer are looking backwards to
just at the beginning. traditional folklore collection, and The Dynamics is an
• Many students reported some initial confusion as example of the new trend, so the latter couldn’t
to what “the lore was often more studied than the possibly be there to “refute” the former. In any case,
folk” meant. LSAT authors will often toss in a lines 18–21 act as a transitional sentence clearly
figure of speech like this—simply to see whether separating the two details.
it throws you! Of course, the author immediately (C) Wrong paragraph. Repertoire analysis doesn’t show
adds “That is,” (line 3)—indicating that the up until paragraph 3.
phrase is about to be redefined. (And, of course, (D) may be highly tempting to less than careful readers,
this is all something of a curve, because the but paragraph 2 is solely about the new interest in
author spends more time on the role of women women folklorists. It sidesteps the “performer vs.
than on the folk vs. lore stuff anyhow.) material” distinction that’s raised in paragraphs 1 and
4 and highlighted in (D).

4
Section I: Reading Comprehension

10. (C) (E) The male/female distinction regarding the meaning


The folklorists mentioned in the last paragraph are of a piece of folklore was not one made by early
concerned with studying folklore in its larger cultural folklorists. This is a distinction made by contemporary
“context,” and in understanding the different ends to scholars.
which it is put by men and women. The book in (C)
reflects these two interests. 13. (A)
(A) overlooks the male/female distinction made by Before the early 1970s, folklorists studied folklore but
current folklore researchers. paid no attention to the people who were involved in
transmitting it. Likewise, the anthropologist in (A)
(B) gets the male/female distinction right, but misses
studies “implements” (i.e., tools) but pays no attention
the cultural emphasis of current scholarship.
to the people who used those implements.
(D) reflects the historical emphasis on the substance of
(B)–(E) None of these choices reflects the basic
folklore itself to the exclusion of those who transmit it.
pattern of pre-1970s scholarship on folklore: an
(E) distor ts a detail from the wrong paragraph— emphasis on a “cultural product” combined with a lack
paragraph 1. of interest in those who actually “use” the product.

11. (B) 14. (D)


This choice captures the essence of lines 7–10. According to the last paragraph of the text, current
(A) Folklore research has recently been concentrating folklore scholarship is concerned with analyzing the
on the differences, not the similarities, in the ways that links between specific pieces of folklore and the larger
men and women use folklore. society from which it emerged. Hence, “context” here
(C) This choice distor ts a detail from the wrong refers to the “environment” and “circumstances”
paragraph—paragraph 3. surrounding a piece of folklore.
(D) Au contraire. Folklore research has increasingly (A), (C), and (E) are all beyond the scope of the
sought to place folklore in its proper cultural context. passage, which doesn’t deal either with the internal
relationships among different parts of a piece of
(E) Since the early 1970s, there has been an
folklore or with the connection between folklore and
increasing scholarly interest in individuals who transmit
physical locale.
folklore.
(B) The text doesn’t delve into the issue of successive
12. (A) interpretations of the same piece of folklore by different
scholars.
Early folklorists, remember, concentrated on the
folklore itself, and not on the people who transmitted
15. (B)
the stuff. Why? Because these folklorists felt that the
performers were not creative people who contributed Abraham’s book is “notable” (line 43) but “unfortunately”
to the material itself. (line 46) lacks a key element, and that amounts to less
than 100% endorsement.
(B) is beyond the scope of the passage, which discusses
scholarship about performers of folklore, not the views (A) The “approval” cannot be “wholehearted” given
of the performers themselves. lines 46–48.
(C) is also beyond the scope of the passage, which, (C), (D), and (E) all ignore the reference to Abraham’s
again, focuses on scholarship about folk performers. book being “notable.” (Moreover, they’re all so close in
There’s no real discussion about folklore itself, let meaning that it’s almost impossible to choose among
alone whether it underwent change from generation to them.)
generation.
(D) distorts the gist of lines 13–18, which state that
early folklorists tended to downplay the contributions of
women performers. That’s quite different from saying
that early folklorists thought that women didn’t have
much of a role in transmitting folklore.

5
PrepTest 15 Explained

Passage 3: Pocock and Political Discourse The Questions:

16. (C)
Questions 16–21
This choice neatly captures the author’s topic, scope,
Topic and Scope: Pocock’s approach to political and purpose; and it clearly echoes the last sentence of
discourse; specifically, his use of linguistic analysis to the passage in its critical approval of Pocock’s work.
interpret the political discourse of the past. (A), (B) “Civic humanism” (A) and “eighteenth-century
political texts” (B) are certainly prominent aspects of
Purpose and Main Idea: The author’s purpose is to
this passage. But the main focus of this text is on
describe and critique Pocock’s methodology. His
Pocock’s methodology and its application to
specific main idea is that Pocock’s methodology is a
eighteenth-century political texts. Yet neither of these
valuable tool for understanding past political discourse,
choices even mentions Pocock.
even if it doesn’t fully explain each and every historical
document. (D) concentrates on a detail that emerges only in
paragraph 3.
Paragraph Structure: Paragraph 1 describes Pocock’s (E) plays on a detail in paragraph 1.
basic methodology and compares it (favorably) to
traditional methods of interpreting political texts of the 17. (C)
past. Paragraph 2 describes Pocock’s work: how he
The association of English whigs and “a vocabulary of
has applied his methodology to investigate “civic
economic progress” is explicitly made in lines 32–34,
humanism” in eighteenth-century England.
but this passage is so dense that you are perhaps to
Paragraph 3 continues the discussion of Pocock’s be forgiven if you missed it.
work, noting that his analysis of political discourse in
(A) Tempting if your outside associations of Jefferson
eighteenth-century America doesn’t ring as true as his
label him as a populist or “progressive,” but we’re
analysis of English political discourse. Nevertheless,
specifically told that, to Pocock, Jefferson echoed Tory
the author ends the passage by saying that, although
vocabulary (lines 42–45). The author disagrees, of
Pocock’s work isn’t entirely on the mark, his
course, but the focus of the question is Pocock’s view.
methodology is certainly on the right track.
(B) To make this choice work you have to go to way too
The Big Picture: much trouble. Once you hear Pocock label Jefferson a
• The passage begins unpromisingly. Take the closet Tory (lines 42–45), you have to assume that
opening sentence, for instance: Many examinees’ Jefferson’s opponents, the Federalists, were therefore
heads were swimming as early as line 7. Note, closet Whigs, and then assume that the Federalists
too, the sleight-of-hand in terms of topic and must therefore have adopted the Whig political
scope: Pocock disappears from the passage until vocabulary mentioned a paragraph ago. Yikes! Far less
line 22, yet turns out to be the central figure. This risky to grab the explicit right answer in lines 32–34.
kind of thing is very rare in LSAT passages. And (D), (E) Au contraire. The Tories are cited as the
the passage stays about as difficult as it begins. antithesis of the Whigs, and the rural landowners are
• The answer, of course, is to get a lot of mileage identified with the former. Both groups’ vocabulary,
out of the other three passages—which do turn inferably, is that “of public spirit and self-sufficiency”
out to be more manageable—and try to budget (lines 29–30), not “economic progress.”
your time so that you end up having more than
the average 8 mins. or so when you finally take 18. (D)
this passage on. To the author, “Pocock’s ideas have proved fertile”
(lines 39–40), and thus it is fitting “to applaud the
historian [that’s Pocock, in case you have fallen asleep
by the end] who” (lines 59–60) while not 100%
successful, has made political vocabularies a vital
topic. The author’s attitude, then, is much more
positive than negative.

6
Section I: Reading Comprehension

(A) 1/2 right, 1/2 wrong. “Fruitful” is positive enough 21. (A)
as a characterization of one of Pocock’s key Paragraph 1 describes Pocock’s basic method of
assumptions, but “cant” (line 39) is Namier speaking studying political discourse; paragraphs 2 and 3
of political language in the 1700s, and is way too describe the application of this method to the cases of
negative to ascribe to the author as a judgment on eighteenth-centur y England and America; and
Pocock. paragraph 3 evaluates the merits of Pocock’s method
(B) “Sharp” is positive, but as used here it describes in light of its application to these cases. (A) reflects
a “contrast’ (line 16), not a characteristic of Pocock. this sequence.
And “elitist” at line 46 is Jefferson’s characterization (B) The author has reservations about Pocock’s work,
of English Tories. and he states them after a presentation of the evidence.
(C) 1/2 right, 1/2 wrong, but in reverse order. (C) What hypothesis? This passage describes and
“Controversial” is applied to some of Pocock’s evaluates a scholarly mode of inquiry.
theories, but the “naiveté” is that of the theorists who
(D) The author does evaluate Pocock’s work and does
preceded Pocock.
suggest a future direction for research, but this choice
(E) “Importance” sounds okay, but in line 55 it’s used says nothing about all of the description of Pocock’s
in connection with different eighteenth-century political work.
vocabularies, not Pocock. “Simply” doesn’t have either
(E) What comparisons and contrasts? What categories
positive or negative overtones.
of evaluation? What framework?
19. (B)
This choice nicely paraphrases the author’s criticism
(lines 17–21) of the method of political analysis used
in the 1950s.
(A) The keyword “while” (line 13) indicates that the
author contrasts the assumptions of the 1950s with
the way literary historians derive the meaning of a
political text.
(C) is beyond the scope of the passage, which never
suggests that political texts can be “read in…different
ways” depending on one’s philosophic bent.
(D) This is an assumption made by Pocock, not
analysts of the 1950s.
(E) The author is critical of the 1950s methodology for
assuming that historical knowledge wasn’t relevant to
the interpretation of documents.

20. (D)
In lines 41–54, the author argues that Pocock erred in
applying the same “linguistic dichotomy” to both
England and America. What works for England, the
author asserts, doesn’t necessarily work for America.
(A) is beyond the scope of the passage. Pocock
concerns himself with political discourse only.
(B) It’s the author, not Pocock, who “denigrates” the
role of analytic philosophers in analyzing political texts.
(C), (E) The author wholeheartedly endorses Pocock’s
interpretation of eighteenth-century political discourse
in England.

7
PrepTest 15 Explained

Passage 4: Black Economic Progress The Questions:

22. (E)
Questions 22–27
Title VII prohibits all employers “from making employment
Topic and Scope: Black economic progress; specifically, decisions on the basis of race.” Executive Order 11,246,
the effects civil rights laws and educational on the other hand, concerns only government contractors.
opportunities have had on Black economic progress. In other words, Title VII “governs hiring practices in a
wider variety of workplaces.”
Purpose and Main Idea: The author’s purpose is to
(A), (B) It’s Executive Order 11,246 that calls for
counter an argument regarding Black economic
monitoring employers to ensure minority representation
progress. Her specific main idea is that civil rights
(A) and deals with government contractors (B).
laws, not increased educational opportunities, account
for recent Black economic progress. (C) and (D) are beyond the scope of the passage, which
never brings up the issues of wage discrimination (C) or
Paragraph Structure: Paragraphs 1 and 2 are minority representation in government (D).
descriptive in nature. Paragraph 1 outlines the relevant
civil rights laws, while paragraph 2 explains the 23. (B)
“continuous change” hypothesis, which argues that Lines 30–32 state that, in the mid-1940s, Blacks were
Black economic progress should be attributed to catching up to Whites in amount of schooling, but were
improved educational opportunities rather than civil not yet equal. (B) makes precisely the same point in a
rights laws. different way.
Paragraphs 3 and 4, to the contrary, are argumentative (A), (C), and (D) are all beyond the scope of the passage.
in nature. In paragraph 3, the author cites several There’s no mention of White school expenditures (A),
reasons to explain why educational opportunities can’t Black or White school curriculums (B), or the general
account for Black economic progress. In paragraph 4, quality of White schools (D).
she claims that the economic progress made by Blacks
(E) 1/2 right, 1/2 wrong. Lines 34–35 say that teachers
since the mid-1960s is directly attributable to the civil
at Black schools did experience wage increases in the
rights legislation of the mid-1960s.
mid-1940s, but we aren’t told whether wages increased
The Big Picture: at a greater or lesser rate than wages for teachers at
White schools.
• Not a promising place to begin work on the
Reading Comp. section. Why? The author’s voice 24. (C)
doesn’t enter the picture until paragraph 3. The
first couple of paragraphs bombard you with a lot The author’s specific main idea, as we’ve already
of detail without giving you any clear signals mentioned, is that Black economic progress should be
about where the passage is headed. In general, attributed to civil rights legislation, not to better
it’s best to work first on passages where topic, educational opportunities. In making this argument,
scope, and authorial purpose are clear early on. she rebuts the “continuous change” hypothesis, which
These passages lend themselves to scoring quick asserts just the opposite.
and easy points. (A) The author contends that the “continuous change”
• When a passage contains a lot of details, it’s hypothesis is incorrect, not that it’s incomplete.
easy to get bogged down in them. Don’t let this (B), (D) The author does indeed discuss both the
happen to you on test day. Remember, you’re only “impact of education” (B) and “Black economic progress
going to get questioned about one or two details. before and after the 1960s” (D), but these are mere
As you read through the passage, move past details intended to support her larger argument.
details quickly, noting where they appear, so that (E) The author provides her own perspective about the
you can look them up easily should you have to. factors behind Black economic progress, not the
“current view.”

8
Section I: Reading Comprehension

25. (C)
This choice neatly summarizes the essence of lines
12–16.
(A), (D), (E) All we’re told about continuity theorists is
that they don’t believe that legislation accounts for
recent Black economic progress; we aren’t told
anything regarding their general attitude about
legislation’s effects on discrimination.
(B) Continuity theorists claim that Blacks have made
progress in education, but they don’t ascribe this
progress to the law.

26. (A)
This concession is sandwiched between information
intended to bolster the claim that civil rights laws have
contributed to Black economic progress. Hence, its
meant to strengthen that claim by explaining away a
possible objection to it.
(B) The only cause of Black economic progress that
the author cites is civil rights legislation.
(C), (D) The author concedes nothing to the continuity
theorists (C). Nor does she alter her argument in light
of their hypothesis (D).
(E) This is scope of the entire passage, not just the
lines that come after 60.

27. (D)
The “continuous change” hypothesis claims that Black
economic progress is unrelated to government actions.
Similarly, (D) features a scenario in which progress is
made without government assistance.
(A), (B), (C), (E) In each of these scenarios, a problem
is solved through government intervention.

9
PrepTest 15 Explained

SECTION II: (E) To “rest on a contradiction” means to base your


argument on two premises that cannot both be true.
LOGICAL REASONING The chemist does contradict, or take issue with, the
physicist, but there are no conflicting premises within
1. (A) the chemist’s argument.
By providing a rationale for prohibiting an otherwise
legal activity—namely, harm to which others must be 3. (E)
exposed—(A) justifies Walter’s recommendation for The argument proposes a cause-and-effect relationship:
the airlines. Smoking is that otherwise legal activity, Because ulcer patients possessed this bacteria strain,
and the nonsmokers on a plane are the “others” who and someone accidentally ingested the strain and got
can’t avoid harm. his first ulcer, therefore the strain probably brings on
(B) “Only if” is the tipoff here. Walter’s purpose isn’t ulcers. Hard to prove, but easy to rebut: If one were to
to describe a precondition for a ban on smoking, but to find test subjects who possess the bacteria strain but
recommend a ban in one specific situation. Even if (B) no ulcer, that would certainly toss the theory out the
matched up well to ever y element of Walter’s window, wouldn’t it? Now, (E) found no such test
argument—which it doesn’t, because the reference to subjects out of a large sample: Many people without
“most situations” has no connection to what Walter ulcers were examined and no traces of the bacteria
says—(B) still wouldn’t justify his proposal, but would strain were found. Hence, (E) supports the reasoning by
simply establish a condition necessary for a smoking short-circuiting a major possible exception.
ban. Think of it in formal logic terms. The stimulus essentially
(C) The “legal activity” would have to be smoking, but argues that “If you have the bacteria strain, then you’ll
how would one “modify” it? By using a cigarette holder? get an ulcer,” the contrapositive of which is “If you don’t
C’mon. have an ulcer, then you won’t have the bacteria strain.”
(D) is a justification for keeping smokers out of planes, And there they are in (E), 2,000 people who fit that
not for keeping planes free of smoke. definition—no ulcer, and no bacteria either.
(E) is a justification for making an activity, presumably (A) A secondary disease is outside the scope of an
smoking, “legal in all situations.” This one is way off. investigation of whether a particular cause results in a
particular effect.
2. (D) (B) The stimulus doesn’t argue that the bacteria
“…your challenge is ineffectual, since you are simply causes only ulcers, so the presence or absence of
jealous…” Those nine words are all you need to answer other ailments is irrelevant.
this “logical flaw” question, and having read them you (C) Even if we accept that one can learn about human
should have jumped to the choices and grabbed (D). diseases from studying other animals—a notion that
One is never permitted to rebut an opponent by comes from without the stimulus, not from within—the
criticizing his or her personality, character, or motives. absence of ulcers in (C) would weaken the case for the
Doing so is called an ad hominem attack, but LSAC causal agent, not strengthen it.
doesn’t expect you to know the Latin term; (D) is how (D) appeals to an expert’s authority. Phooey. Experts
the testmakers typically word it. can be wrong.
(A) The chemist restates no claim, and the use of the
Evidence Keyword “since” establishes that he does 4. (D)
offer evidence. It’s not effective evidence, but it’s The stimulus conclusion—to which we seek a parallel
evidence nonetheless. —is hypothetical: Should one of these test subjects
(B) is only tempting if you decide that since the physicist own a pet, he or she would have a lower average blood
believes that “measurements and calculations are pressure. The evidence? Lowered blood pressure
inaccurate,” the chemist must believe that they are apparently caused by petting animals. (D) has it right.
accurate. But as we’ve noted, the chemist disagrees (D) uses the relaxing caused by a short boat ride in the
with the physicist on personal grounds, not on the same way that the stimulus uses the lower blood
grounds of accuracy. pressure caused by animal petting: as evidence for the
(C) Nope, in this context “solve” has only one meaning: hypothesis that ownership of an object (a boat and a
to locate the answer to a problem. pet, respectively) would cause a general improvement
of the trait (relaxing and lower b.p., respectively).

10
Section II: Logical Reasoning

(A) is only superficially similar to the stimulus. Its evidence concludes that Namibia’s landscape must have
lacks the sense of one action having a particular effect changed over the centuries, but that assumes that that
on one limited group of people, and its conclusion fails ape did, in fact, live in or near Namibia—choice (B). If
to allege that ownership of an object would have a it didn’t—if it wandered into Namibia far from its
greater effect. forested homeland and thereupon expired, or if
(B), (C) Each errs by focusing on a policy recommendation perhaps the jawbone was blown by the wind or carried
—retaining a car and dumping cer tain plants, by someone to that spot—then Namibia’s terrain need
respectively. The stimulus never recommends anything; not have changed at all. Since (B), if false, would
it predicts what would happen (lower blood pressure) if weaken the argument, it is a necessary assumption.
the test subjects owned a pet. (A) Modern apes are outside the scope of this
(E), like (A), may be tempting because of its hypothetical argument. It’s modern Namibian terrain and ancient
conclusion. But in the stimulus, petting an animal and apes that are at issue.
owning a pet are two different things—which is in fact (C) Certainly the author believes that at least one
the root of the flawed logic: Since they are different ape—the one whose jawbone was found—lived in
things, one cannot assume that the salutary effect Namibia between 10 and 15 million years ago. But
caused by petting an animal would be repeated, let apes prior to 15 million years ago are outside the
alone intensified, by pet ownership. But (E)’s one coat scope.
vs. two coats lacks that difference. Two coats of paint (D) Why apes lived in dense forests may or may not
probably would make for a whiter fence. have anything to do with their diet. And (D), like (A),
errs by mentioning modern apes.
5. (A)
(E) provides an explanation as to how the change in
This discrepancy (or “paradox”) hinges on the definition Namibian terrain might have taken place, but it’s far
of “best bill collector.” How can he be the best if his from the only possible one. In any event the author is
collection rate is the worst? Maybe you were able to arguing about whether the change occurred, not how.
predict the answer and maybe you weren’t, but we
hope you recognized it as soon as you saw it: The 7. (B)
reason his collection rate is so bad is that he’s
Is job-related stress the #1 workplace problem? Our
assigned to the hardest-core cases—as would befit the
author concludes no, it’s not, because most workers
best employee in the firm.
complain about boredom, not job stress. But if stress
(B) is akin to (D) back in Question 3. What Young’s co- and boredom are somehow related, then there’s no
workers think of him is as irrelevant to this paradox as contradiction, and that’s essentially what we get in (B).
the expertise of the stomach researcher is to the (B) implies that workers who explicitly complain about
cause of ulcers. boredom are implicitly demonstrating signs of stress;
(C) deepens the paradox, by suggesting that his crummy hence the two go hand in hand; hence stress is
rate of collections is in fact his norm. probably the #1 problem.
(D) is laughably irrelevant—though if you chose (D) (A), (D), (E) Non-complaining workers are outside the
you’re probably not laughing. We’re not told whether scope here, since the issue is “What’s the most
Young’s credit dept. job involved collections, and if it serious workplace problem?,” and the source of the
did, whether Young was good or bad at it. Not that any evidence is those who are making their complaints
of that would make any difference to his performance known. Workers who are relatively happy, for whatever
at this job. reason, aren’t part of this debate.
(E) Length of tenure is irrelevant too, having nothing to (C) doesn’t discuss complainers, either—just responders.
do with Young’s collection rate belying his status as the And the “recentness” of responses has nothing to do
best collector. with the argument.

6. (B) 8. (A)
The author sees a bit of a paradox here: Ancient Keywords are the key to structure, and structure is the
primates, such as the one whose jawbone was found key to figuring out this author’s conclusion quickly.
in Namibia, lived in dense forests, yet today Namibia is A yes/no question is posed: Should the government
dry and treeless. To resolve this paradox, the author stop trying to figure out how much toxin is O.K. in our

11
PrepTest 15 Explained

food? Three potent Keywords govern what’s left, (A) Nothing is ever said about workers’ spending habits
starting with “Only if,” which as always signals a or abilities.
necessary condition—necessary, that is, for a Yes (B) Job creation is never mentioned.
answer to the question. “However” signals a contrast
(C) The percentage of all persons who work is never
—i.e. a statement that that necessary condition has
even alluded to.
not been fulfilled. And “furthermore” is a Continuation
Keyword, which means “more of the same.” Feel free (D) “Anticipated” by whom? Nothing is mentioned about
to read more deeply into the stimulus if you like, but what was in people’s minds 25 years ago. (And even if
the sheer structure leads to only one conclusion: A “anticipated” is read as a paraphrase of “potentially,”
condition necessary for abandoning the government’s all we’re told is that the number of hours worked was
efforts has not been met, hence the government potentially smaller and the number of leisure hours
should not abandon its efforts—choice (A). (Note that potentially greater. Nothing about output expectations
(A) substitutes “should continue” for “should not whatsoever.)
abandon”—a common testmaker tactic.) Sherlock Holmes had it right: When all other possibilities
(B) is the “Only if” clause—the condition necessary, in have been eliminated, what’s left, however implausible,
the author’s view, for the government to abandon its must be the truth.
efforts.
10. (C)
(C) is a very close paraphrase of the “However” clause
—a piece of evidence suggesting that the necessary Yet another paradox. A bunch of ancient Asian
condition has not been met. communities stopped hunting-and-gathering and
started cultivating their food. Their diet and health went
(D) is an equally close paraphrase of the “Furthermore”
to blazes compared to the old days, and yet they never
clause—more evidence.
went back to hunting-and-gathering. How come?
(E) cannot be the author’s conclusion, because the Pre-phrasing an answer might be tough, especially if
issue is not “Are the government’s detection methods you’re unfamiliar with the archaeological terminology,
refined enough?”; no evidence either way is provided. but we hope that your search of the choices yielded (C)
(E) is a misreading of the very last clause of the stimulus. reasonably readily. If, as (C) suggests, there were just
Inferably the author would probably approve of the too many people to make hunting-and-gathering viable,
government’s raising “the threshold of detection,” but why indeed would those ancient Asians go back to it,
only because of her conclusion that detection needs to notwithstanding their dietary problems?
continue…which brings us back to (A).
(A), if anything, would provide a reason for the Asians
to resume hunting-and-gathering—to grab all that wild
9. (E) flora and fauna.
The stimulus for this inference question is more (B)’s contrast of stored vs. fresh foods has nothing to
complex than most, but all four wrong answers are do with the argument at hand, and in any case
arguably more obvious than most, so perhaps it describes something that hunter-gatherers and
balances out. agriculturalists had in common, which doesn’t do the
Over a quarter-century of labor-saving technologies, job.
we’re told, workers have been able to produce goods (D) rather comically implies that the Asians failed to
(“output”) a lot faster, with the potential result that a return to hunting-and-gathering in order to keep up with
worker could put in fewer hours and get more leisure the Joneses elsewhere in the world. Huh?
time. However, there has not been correspondence:
(E) seems to imply that ancient Asians chose agriculture
The average output per hour has grown twice as fast as
over hunting-and-gathering because the former burned
the average worker’s leisure time. (E) follows from that
more calories and hence would make them buff up. But
last sentence: If the average worker is producing goods
that makes no sense: Remember that the agricultural
a lot faster but isn’t working a lot fewer hours, then he
peoples had a terrible diet and bad health compared
must be producing more per week than he did a
with the old hunter/gatherers. Concern for their well
quarter century ago, prior to the introduction of all
being would seem to mandate returning to the old
those technologies.
ways—which they failed to do. So (E), if anything,
Don’t see it? Check out the lameness of the other four deepens the paradox.
choices:

12
Section II: Logical Reasoning

11. (E) channels; we have absolutely no reason to infer that


Just like question 8, this one begins with a question only a “little” did so.
that the author is apparently going to attempt to
answer. There are only three possibilities allowed by 13. (E)
the author to begin an article with the phrase “in a The author says that the pedigree standards applied to
surprise development”: the development was a working dogs ignore those genetic traits that allow the
surprise to the journalist alone, the development was a dogs to serve the purpose for which they were originally
surprise to some other person, and the development bred; as a result, those practical traits may be lost.
was a surprise to people in general. In each case, the She decides that the standards set by pedigree
author says that the set phrase is inappropriate. The organizations for working dogs should include the
argument’s conclusion, therefore, is (E): In no case is ability to do the work for which they were intended.
it a good journalistic practice to begin a paragraph with We’re looking for a principle that suppor ts this
the phrase “in a surprise development.” conclusion. (E) says that organizations that set
(A) and (B) The journalist never describes circumstances standards for activities or “products” (and presumably
in which it is appropriate to use the phrase “in a pedigree dogs fit this designation) should ensure that
surprise development.” they can serve their original purposes; i.e. according to
(E), the author is right and pedigree organizations
(C) Although the author does describe three distinct
should ensure that “working” dogs are still able to
sorts of circumstances in which the phrase is used,
work.
that’s not his conclusion; he makes that distinction in
order to help him arrive at his conclusion, that in no (A) talks about the sort of standards organizations
circumstances is it appropriate to use the phrase. should not set, but it doesn’t justify the author’s
conclusion about what standards pedigree organizations
(D) This is way off; the author never discusses what’s
should set.
permissible in summing up a story; he only considers
the phrase “in a surprise development” as an (B) The author isn’t in favor of enforcing those
introduction, and even there he finds it wanting. standards currently in effect, but favors setting new
standards.
12. (C) (C) argues that organizations should see that their
Despite the technical jargon, this stimulus is actually standards are met; the author is attacking the prior
made up of a bunch of formal logic statements. Take question of what sort of standards should be set in the
the third sentence: If X (yellow to black), then either Y first place.
(polypyrroles form on zeolites) or Z (polypyrroles form (D) The author doesn’t favor standards requiring working
in zeolites). The last sentence tells us that X occurred ability because today’s pedigree dogs will actually be
(yellow to black), but Y didn’t. From this we can deduce required to work; she argues in favor of the standards
that Z must have occurred: polypyrroles formed from because they reflect the original purpose of the breeds.
pyrroles inside the zeolite. The second sentence tells
us that when polypyrroles form inside the zeolites, they 14. (B)
form in delicate chains. Therefore, we can infer (C): The author is attempting to establish the conclusion
some of the pyrroles in which the zeolite was that standards set by pedigree organizations for
submerged formed into polypyrrole chains. working dogs should require working ability. The phrase
(A) The stimulus says that the zeolite was “free of any “certain traits like herding ability risk being lost among
pyrrole” before it was submerged, so (A) is impossible. pedigreed dogs” is intended to suppor t this
(B) Since the polypyrroles must have formed in and conclusion. The phrase itself is supported, since it
not on the zeolite, they must have been formed as occurs at the end of a chain of reasoning: Pedigree
chains, not lumps. organizations don’t specify work traits among their
standards; breeders only maintain traits specified by
(D) There’s no reason why some pyrrole couldn’t attach
pedigree organizations; traits not maintained by
itself to the zeolite; all we know is that no polypyrroles
breeders risk being lost; therefore work traits like
formed from pyrroles on the zeolite.
herding ability risk being lost. As (B) says, the phrase
(E) Since the polypyrroles formed inside the zeolite, is a “subsidiary” conclusion; that is, it’s a conclusion
we know some pyrroles reached the zeolite’s inner drawn on the way to the main conclusion.

13
PrepTest 15 Explained

(A) On the contrary—as we’ve seen, it’s supported by 16. (E)


several pieces of evidence. The author has concluded that for the amaryllis plant,
(C) Far from acknowledging a possible objection to the going dormant is a positive thing, and that therefore an
author’s proposal, the phrase states the main reason amar yllis plant’s owner should actually induce
for the author’s proposal. dormancy in his plant. But the evidence doesn’t
(D) is dead wrong. The argument accepts the claim support this; it seems much more likely that going
that traits like herding ability risk being lost; the dormant is simply the amar yllis plant’s defense
author’s proposal is intended to avert this very real against the drought that occurs in its natural habitat. If
danger. so, then without a drought there’s probably no need for
a period of dormancy. As (E) recognizes, the author
(E) No earlier claim depends on the phrase; rather, the
assumes that there’s more to dormancy than a
phrase is supported by earlier claims, and in turn gives
defensive reaction, that the dormancy confers some
support to the conclusion.
other positive benefit on amaryllis plants.
15. (C) (A), (B) The author only speaks of amaryllis plants,
and so need not assume anything about other plants or
You may not have been able to pre-phrase this
how other plants compare to amaryllis plants.
“strongly-supported conclusion,” but hopefully choice
(C) made sense to you upon evaluation. The arthritis (C) The author never specified when water should be
medication works by inhibiting the functioning of the withheld from amaryllis plants kept as houseplants,
hormone that causes pain and swelling. However, the only that it should be done at some time each year to
hormone doesn’t only cause pain and swelling in cases induce dormancy. The lack of specificity in the
of rheumatoid arthritis; it’s also normally activated in argument regarding the time frame for withholding
response to injury and infection. It therefore seems water kills this one.
reasonable to suppose that the medication would (D) Although the author believes dormancy is good for
inhibit the normal action of the hormone in response to amaryllis plants, he needn’t assume that the only thing
an injury. This in turn supports (C)’s contention that a that could go wrong with an amaryllis plant is too short
person taking the medication might sustain a joint a period of dormancy.
injury and, because of a lack of the pain and swelling
that normally accompany such an injury, be unaware 17. (D)
of it. The author accepts the theory that yawning is most
(A) There’s no evidence that the medication would powerfully triggered by seeing someone else yawn
repair cell damage that has already occurred; quite the because 1) a lot of people believe the theory today and
contrar y, since the only action ascribed to the 2) according to “historians of popular culture” people
medication is that of inhibiting the functioning of the all over the world have commonly believed the theory.
hormone. That’s not a very convincing argument; we’d like to
(B) No mention of harmful side effects is made in the have something more than widespread belief to fall
argument, so it’s impossible to know whether or not back on. That’s the flaw (D) points out; the argument
any side effects would be outweighed by benefits. relies on opinion, whereas the question under
consideration—“what triggers yawning?”—is a
(D) We have no idea whether the hormone inhibited by
scientific, or factual question, the answer to which
the medication has anything to do with diabetes or
needs to be suppor ted by substantive factual
lupus, so there’s no reason to conclude that the
evidence.
medication would be of any use in combating these
diseases. (A) The author doesn’t indulge in circular argumentation
—his evidence, albeit weak, is certainly different from
(E) On the contrary, we’d expect the medication to have
his conclusion.
some effect on any joint disease involving the production
of the hormone that causes pain and swelling. We (B) The author never defines “historians of popular
don’t know that there are any other such diseases culture,” but there’s no reason to suppose that popular
besides rheumatoid arthritis, but we certainly can’t beliefs about yawning are outside their area of
conclude that there aren’t. expertise.

14
Section II: Logical Reasoning

(C) The author refers to common popular belief; he opposition leader spilled the beans—just as likely as
makes no reference to particular cases, either typical the author’s conclusion.
or atypical. (B) The author never states that the chain of events
(E) The author concludes that seeing other people yawn was inevitable; that is, he never assumes that, once
is the “most irresistible” cause of yawning; he never the information from the secret meeting was released,
claims, or assumes, that it’s the only cause of yawning. it was inevitable that the minister be brought down.
(C) The author never discusses any “different occasion”;
18. (B) both evidence and conclusion concern this particular
The stimulus in this Parallel Reasoning question is so finance minister and this secret meeting.
formal in nature that it can be boiled down to algebra: (D) The evidence is entirely relevant to the point at
All A are B (All gourmet cooks enjoy variety). issue, which is “who was responsible for bringing the
No B is C (No one who enjoys variety prefers bland finance minister down?”
food). (E) The evidence is that the effect (the minister’s
Therefore, no C is A (or if you prefer, All C are not A, downfall) was impossible without the action (giving the
same difference: The conclusion is that gourmet cooks information to the newspaper). This means that the
and preferrers of bland food have nothing in common). latter was necessary for the former; the author never
treats the evidence of the action as though it was
In correct choice (B), Huang Collection paintings are A,
sufficient to bring about the effect.
abstract paintings are B, and paintings included in next
week’s auction are C, and if you plug those terms into
20. (A)
the algebra above, you’ll see that (B) functions
identically. Here’s how the wrong choices play out: Evans discounts those critics who dismiss him and his
poetry, because he says they themselves are not true
(A) All A are B. All B are C. Therefore, all A are C. This
poets and only a true poet can function as a critic of
is the classic Aristotelian deductive syllogism, by the
poetry. Why does he say that they’re not true poets?
way. The sheer absence of a negative or “not” term
He’s read their poetry and doesn’t think much of it—it
means that you could have rejected (A) right away.
doesn’t “convey genuine poetic creativity,” as true
(C) All A are B. No A is C. Therefore, no C is B. Note poetry should. He’s relying on his own judgment of
the second premise’s subtle shift away from the poetry; therefore, he’s assumes that he’s a fit judge of
original. poetry and poetic creativity, and according to his own
(D) All A are B. No C is B. (So far so good, actually. criteria, he must also be assuming that he is a true
Remember that No B is C is the same as No C is B. poet. As (A) points out, he’s assuming exactly what he
They’re reversible.) Therefore, no B is C. (Oops; close set out to prove, that the critics are wrong and he’s a
but no cigar.) true poet.
(E) All A are B…and here algebra falls short, because (B) Evans never assumes or implies that everyone is
the remainder of (E) brings in complex terms like either a poet or a critic. All she says is that if you’re not
prices in general, adequate reflection of price, true one, you can’t be the other.
value, and next week’s auction. Because (E) is not as (C) Evans never implies that a poet’s standing can be
strictly written as the stimulus, it, like (A), can be judged independently of his or her poetry; in fact, the
quickly rejected. criteria given for a true poet is one whose works
conveys genuine poetic creativity.
19. (A)
(D) Far from making an absolute distinction between
Because no one witnessed the meeting between the critics and poets, the author says that only true poets
minister and the opposition party leader except for the can function as critics of poetry.
minister’s aide, the author concludes that it must have
(E) On the contrary, true poets would be in a position
been the minister’s aide who provided the information
to criticize their own work.
that brought the minister down, rather than a political
enemy of the minister. Does this make sense? After
21. (E)
all, the opposition leader was at the meeting too;
mightn’t he or she have given the information to the Ouch!—one of the nastiest question stems we’ve
newspapers? As (A) points out, the evidence provided come across in a long time. If there was ever a
by the author, makes a competing conclusion—that the question worth blowing off temporarily strictly based on

15
PrepTest 15 Explained

its stem, this is it. Be that as it may, let’s take the (A) completely ignores the issue of food availability
stem apart to see what they’re after here. which is the whole basis of the theory; the scientist
The correct choice will be able to serve as the premise said nothing about temperature, so (A) is irrelevant.
of an argument against the claim, but it must be an (B) introduces an irrelevant distinction; the scientist
argument that relies on the principle. So we’re not was interested in comparing the total amount of
looking for something that by itself weakens the claim, nourishment available in the different places. The point
but rather the basis for an argument that, if based on is that fish need a lot of nourishment when they mature
the principle, would weaken the claim. Tricky, eh? and little when they spawn—it doesn’t matter what
According to the principle, if one claims that Country X type of food they eat to get that nourishment.
lowered its taxes to serve the advantage of foreign (C) Because the scientist is talking about general
companies, one must show how the foreign migration patterns, it doesn’t matter if there are a few
companies’ interests actually helped bring about the exceptions to the rule. Thus, if there were a species of
change; that is, one must show that the reason for the fish with populations in both areas, it wouldn’t make any
tariffs’ being lowered was the usefulness of that difference to the theory whether the fish exhibited the
course of action to foreign companies. Since we’re same or different migration patterns in the two different
supposed to counter the claim that Country X’s tariffs zones; what matters is what fish “generally” do.
were lowered in order to help foreign companies, we
(D) is pretty plainly irrelevant. The total number of
should therefore look for a choice that weakens the
species has nothing to do with the role of food in
connection between foreign companies and the change
determining migration patterns.
in policy. The best choice is (E); if there’s no evidence
that foreign companies themselves played a role in
23. (B)
inducing the change then, according to the principle, it
becomes much more difficult to explain the change by According to the author, computers can only solve
the fact that it served their interests. problems by following some set of mechanically
applicable rules, and some problems just can’t be
(A) The principle says nothing about the interests of
solved that way; it follows therefore that no computer
any second group, like consumers, being served, so
can solve that type of problem. But the author takes
(A) doesn’t apply to the principle.
this one step further; she says that no computer will be
(B) introduces new economic considerations like able to do everything that some human mind can do.
importers’ profits; the principle had nothing to do with She must be assuming that some human mind can do
such things, only with who was responsible for inducing what she’s just shown no computer can do—that is,
the change. she assumes that some human mind (or “at least one”
(C) discusses the short-term effects of the change on human mind, as (B) says) can solve one of those
Country X; we’re only interested in whether the change problems that can’t be solved by following a
occurred in response to foreign interests. mechanically applicable set of rules.
(D) is better than (C) because at least it tells us about (A) The author needs to establish that some non-
foreign companies, but it doesn’t tell us what we mechanical problems are solvable by humans; it
wanted to know—whether they helped bring about the wouldn’t help to assume that there’s some non-
change. mechanical problem that’s UNsolvable by humans.
(C), (D) and (E) all make the same mistake. Since
22. (E) computers also have the ability to solve problems by
The scientist discovers that in the tropics, migratory following mechanically applicable rules, human ability
fish generally follow a migration pattern that’s exactly to solve such problems doesn’t show that humans can
the opposite of the pattern followed by fish in do something that computers can’t. It wouldn’t help
temperate zones: they mature in fresh water and the author to assume that one such problem can be
spawn in the ocean. Does this disprove the scientist’s solved by every human (C), that every such problem (of
theory that food availability determines migration a certain sub-type) is solvable by almost every human
patterns? That’s impossible to know, unless we know (D), or that every such problem is solvable by some
something about the food availability in the tropics—is human (E). We’re only interested in human ability to
there more or less food available in the ocean? Unless solve problems that can’t be solved by following sets of
we know the answer to (E), we don’t know whether the mechanical rules.
fish in the tropics are acting in accordance with the
scientist’s theory or in violation of that theory.

16
Section II: Logical Reasoning

24. (A)
The author is trying to demonstrate that the typical
response given to the survey is ambiguous. He does
this by taking that typical response and applying it to
the case of a theoretical representative individual. Take
a 48-year old man who says he feels as he did when he
was 36; at the age of 36, if he had made the same
type of response that he does now, he would have
claimed to have felt as he did when he was 27; and so
forth. This is an odd turn of events. How did the author
get from the group results to reapplying the typical
response over and over to a single person? As (A)
says, the author projects from the survey response to
hypothetical responses made by a single individual; he
looks at the responses the individual would have made
at an earlier age, and then another earlier age, and so
on, and uses that to show that the current response is
ambiguous.
(B) The author never discusses what would have been
the “most reasonable” thing for the respondents to say.
(C) The example of the 48-year-old man isn’t used as
a counterexample; it doesn’t show that people really
didn’t “almost unanimously” respond by saying that
they felt 75% of their real age. Instead, the example is
used to show that the response, which the author
never doubts, is inherently ambiguous.
(D) The author doesn’t set up two opposed statements
in direct contradiction of each other and force us to
choose one or the other; he’s simply interested in
showing that it’s difficult to understand what one
statement means.
(E) The author never speaks of “manipulation” on the
part of the questioners.

17
PrepTest 15 Explained

SECTION III: (D), like (A) and (B), misunderstands the argument as
an attempt to judge literary quality.
LOGICAL REASONING
3. (C)
1. (B) If you recognize the president’s subtle but definite
The stimulus begins with a claim made by scope shift, you can knock this question off in a jiffy.
Shakespeare-philes, who feel that in England, at least, All the evidence shows is that the material used in the
Shakespeare’s work has been known and loved by all mailings is recycled. But the conclusion is that the
classes, not just the elite. The key phrase comes mailings are recyclable—which is different. The
midway: “Skepticism about this claim is borne out president must be assuming that that which has been
by…” This indicates that the author’s main point is to recycled can be recycled again.
take issue with that claim. Meanwhile, the (A) The conclusion deals with the recyclability of the
counterevidence comes from only one period and mailings only. Paper put to other use is outside the
source, the bound copies of Shakespeare from the scope.
early 1800’s that the author feels could not have been
available to ordinary folk. So (B) is correct in taking (B) The Kaplan “Denial Test” shows you why (B) is
issue with the original claim, and has an appropriately wrong. Suppose for the moment that (B) is false:
narrow scope (“at some time in the past”). Suppose some of the group’s mailings lack windows.
Would that new fact affect the argument? Not a bit; the
(A) The issue here is ‘Who historically has revered mailing materials could still be completely recyclable.
Shakespeare (the elite or all classes?),’ and not ‘How In contrast, note that if (C) is false—if the window
can you tell the elite apart from everyone else?’ material isn’t recyclable—then the argument falls
(C) What aspects? What lack of appreciation today? apart.
(D) goes too far. The evidence, as noted above, comes (D) As anyone who has discarded newspapers or cans
only from the 1800’s. The author cannot be leading to in the normal trash will attest, that which is recyclable
a blanket statement that only the educated upper class isn’t always recycled! The issue here is recyclability,
has ever known and loved Shakespeare. which has nothing to do with what actually happens to
(E) It’s not the elite who are skeptical but the author, the mailings when they arrive at people’s homes.
and that skepticism is of the claim made in sentence (E) The conclusion concerns only “these mailings”—
1, not of whether Shakespeare in fact wrote worthy the ones sent from HQ. That other mailings may or may
plays. not be sent from elsewhere is beside the point.

2. (E) 4. (A)
Even if only the educated rich of the early 1800’s could The first line of attack is to mentally rewrite the first
own and peruse the books that the author cites, who’s sentence. (And paraphrasing a key idea is always
to say that everybody else couldn’t have known, smart anyway.) If that “frequently expressed view…is
studied, and loved Shakespeare by seeing his plays false,” it must mean that, to the author, written
performed? The unwarranted assumption that people constitutions are inherently no more liberal than, and
could only get to know Shakespeare’s plays in book may be even less liberal than, unwritten ones. Which is
form drives a stake through the heart of the logic. all (A) is saying. Notice that the actual “therefore”
(A), (B) The conclusion drawn, which lest we forget is sentence, the one you expect to be the conclusion,
(B) in Question 1, isn’t aesthetic—“literary quality,” to does in fact simply echo sentence 1: It asserts a
quote choice (A)—but sociological, on the topic of condition (liberal interpretation and application) that’s
‘Who has always revered Shakespeare?’ And contrary necessary for a written constitution to become liberal,
to (B), the evidence isn’t “purely economic”—it in contrast to the view that a written constitution is per
connects money to awareness of Shakespeare. se liberal.
(C) is way off. No 20th century standards are cited, and (B) screws up the phrase “properly understood.” The
it’s not 18th century events that are being judged or author uses it in sentence 3 to introduce her view of
evaluated but a contemporary claim about the nature what a constitution should be properly understood to
of Shakespeare’s perennial fans. be—namely, a sum of procedures. But (B) crazily
implies that one cannot properly understand a written
constitution. Huh?

18
Section III: Logical Reasoning

(C), besides making no sense (how could it be easier 6. (E)


to misinterpret a written document than a set of The author recommends that scientists focus on Earth-
concepts that are mental or verbal only?), falls into like planets and carbon-based life when estimating the
much the same trap as (B), by implying that the issue probability of life on other planets, even though there
is how a constitution can be understood or interpreted, could well be non-carbon-based life on planets unlike
when the issue is, in fact, How liberal per se, is a ours. How come? Because all known life forms are
written constitution? carbon-based, and only Earth, to our knowledge,
(D) The preservation of constitutions is never mentioned suppor ts life. So the author is suggesting—for
or alluded to. whatever reason—that we’re wisest to use what we do
(E) There probably are criteria for evaluating how a know in order to make reliable predictions about what
constitution is to be interpreted and applied, but the we don’t. Only (E) explicitly evokes this idea of
author never gets into that. What she’s presenting, “concentrate on the known.”
in the last sentence, are criteria for deeming a (A), as a principle, would support a recommendation
constitution as “liberal.” that we not make use of observed phenomena when
making inferences about unobserved phenomena. This
5. (B) is pretty much the antithesis of the author’s point of
Sentence 2, by itself, yields the correct answer. (B)’s view.
sentiment, that one cannot evaluate the liberalness of (B) sets up a contrast—explaining all phenomena vs.
a written constitution from the writing alone, directly explaining some phenomena—that has no connection
follows from sentence 2’s assertion that words have to to the stimulus whatsoever.
be put into effect or practice before they can be (C), like (A), goes against the text. The author isn’t
assessed. opposed to making estimates about extraterrestrial
(A) contradicts both (B) and the text. By the author’s life, as is the principle in (C); she’s simply in favor of
definition, a paper with words on it is meaningless until basing those estimates on known rather than unknown
those words get turned into action. factors.
(C) makes little sense (it has to be somewhat handy to (D) may sound impressive, but is no closer to the text
have a written constitution to consult from time to than (B) is. The author is in no sense talking about
time), and shifts the scope by implying that the issue “explaining phenomena,” and has nothing to say about
is which type of constitution is preferable, when it’s reliance on a few principles vs. reliance on many.
really about which one is inherently more liberal.
(D) reflects a faulty paraphrase of sentence 1, and if 7. (B)
you picked (D) please read and follow this explanation The politician begins by citing the need for the
carefully. To state, as the author does, that it’s false redistribution of wealth, without which economic
that written constitutions must be more liberal than injustices will flourish. He then predicts that such
unwritten ones, doesn’t mean that the reverse is injustice, if it peaks, will turn those who are suffering
true—that unwritten ones must be more liberal than to violence. Finally, he asserts that the nation must do
written ones. As noted in the explanation to Question whatever it can to alleviate problems—inferably
4, all we can assume is that written ones don’t have an including the economic injustice just described—that
edge when it comes to liberalness. In fact, as we see would otherwise lead to violence. So the implicit point
from the argument as a whole, the author believes that must be: We had better redistribute wealth, choice (B),
neither type of constitution is inherently more liberal lest the chain of causation just described come to pass.
than the other; it all depends on how a constitution is (A) Though the politician believes that violence is
interpreted and applied. inevitable result of intolerant injustice, and probably
(E), besides leaving out the issue of application (which believes that that is regrettable, his purpose is neither
is coequal with interpretation in the argument), to justify nor to condemn—his purpose is to
commits the common error of mistaking a necessary recommend a course of action that will avoid a lot of
condition for a sufficient one. Note that Keyword escalating problems.
“only”! Just because a constitution is interpreted and (C) This can’t be the politician’s conclusion because
applied liberally doesn’t make it liberal—there may be no evidence to this effect is provided; the topic is
other requirements that need to be met before a economic justice. In any event, he apparently believes
constitution is deemed to be a liberal one.

19
PrepTest 15 Explained

that decisions should be based on practical realities (D) Useless background information: So they’re rare,
and not on either of the grounds that (C) mentions. so what? The fact remains that some were observed in
(D) Actually, his point is that economic injustice had 1985 and fewer in 1989, and the issue is the
better be remedied in order to prevent intolerable explanation for that decrease. (D)’s historical note is
social conditions that he believes are an inevitable at most of footnote interest only.
consequence of its continuation. (E) The reference to predators is tempting, if only
(E), like (E) in Question 5, mistakes a necessar y because eaten beetles are nonexistent beetles, but
condition for a sufficient one. “Unless” is a Keyword it’s a dead end. If predators are relevant in this
signaling necessity. Redistributing wealth may not bring context, then why would they be any more or less
about economic justice, but without it, violence is relevant in 1989 than 1985? (E) completely ignores
inevitable. the essential time comparison at the heart of the logic,
so although you may feel that it raises a potentially
8. (A) relevant element, it can’t possibly strengthen the
reasoning.
The data aren’t presented in the most helpful order.
You might have tried to rearrange the facts
9. (B)
chronologically, and made a mental picture of the
situation, too: In 1985, they found 38 of these beetles In concluding that raising blood-magnesium levels can
in a time span of two hours, while in 1989, in the same cure fatigue, the author interprets the obser ved
place, they only found 10 in nine hours. Those data correlations between fatigue (especially fatigue
might suggest a decline in the beetle population over involved in chronic fatigue syndrome) and low levels of
those four years, but no: The author’s conclusion is blood magnesium as indicating that the low levels of
that there probably was no drop because (Evidence magnesium caused the fatigue. But he hasn’t shown
Keyword) ‘85 was wet and ‘89 was dry. Sentence 1 this to be the case. As (B) points out, it’s quite
tells us that these normally motionless beetles move possible that it’s fatigue that causes lower
around more in wet years. What’s the connection concentrations of magnesium in the blood. If that were
between this fact and the data presented? The the case, raising the level of blood magnesium would
implication is that in 1989, despite what the observer simply attack a result of fatigue, not the cause, and
reported, there were pretty much the same number of would be unlikely to effect a cure.
beetles in that spot, but they weren’t moving as much (A) The argument doesn’t rely on the claim that
so they couldn’t be seen. (A) supports the argument malabsorption of magnesium is always the cause of
by shoring up this implication—to escape the gaze of low blood magnesium; the only causal link that plays a
the observer the beetles would have to be pretty role in the conclusion is the link between low blood
darned invisible when motionless. It makes more magnesium and fatigue.
plausible the notion that the obser ver missed (C)’s possibility is irrelevant; the fact that there is
approximately 28 additional beetles that were actually some variation in magnesium levels among healthy
there in 1989. people wouldn’t challenge the author’s belief that
(B) doesn’t necessarily describe the environment at raising the levels of blood magnesium would cure the
which the observer was conducting the study, and in fatigue associated with the syndrome.
any case, other beetle habitats are outside the scope (D) and (E) both criticize the author for not being specific
of this argument, which concerns one locale only. enough, but there’s no reason he should be specific.
(C) mentions reproduction in the expectation that He’s discussing in the most general terms what sort of
you’ll connect it with the issue at hand, which you treatment would cure chronic fatigue; he’s not writing out
shouldn’t: A comparison of the number of beetles in a prescription. Since his conclusion is general, he
two separate years has nothing directly to do with needn’t do the things discussed in (D) and (E).
reproduction activity. But even if they were connected,
(C)’s suggestion that beetle movement relates to 10. (B)
reproduction activity would weaken the author’s logic, The consumer advocate argues that explicit safety
because it would link the dry weather in 1989 to less labels should be required for toys because it would
frequent reproduction and hence support the idea of a enable parents to be more effective in preventing toy-
1989 population drop. related injuries; the labels currently required, “age-
range” labels, don’t do enough to reduce such injuries.

20
Section III: Logical Reasoning

(B) strengthens this argument by confirming the 12. (E)


advocate’s claim that age-range labels and safety The author says that organic farming leaves less land
labels have a different effect on parents. To take the available to serve as habitat for local wildlife than
author’s own example, when parents see that a toy is chemical farming leaves, because organic farming
recommended for ages three and up, they take this to requires that more acreage be cultivated in order to
mean that younger children won’t understand or enjoy produce the same amount of food. The author reasons
the toy; they don’t realize that it might mean that the that the more acreage cultivated by farmers, the less
toy is dangerous for children younger than three. If the land is left as habitat for local wildlife. He must be
label specifically warned that the toy can be assuming, as (E) says, that the land cultivated as
dangerous, parents would presumably be doubly acreage by organic farmers can no longer serve as a
careful that it not fall into their young children’s hands. habitat for wildlife; otherwise, the increase in land
(A) says some toys aren’t dangerous; this doesn’t cultivated wouldn’t constitute a loss of wildlife habitat.
show the need for requiring safety labels for dangerous (A) The author hasn’t discussed the health threat
toys. posed by chemical farming at all; for all we know he
(C) doesn’t strengthen the advocate’s argument because accepts the argument that chemical farming threatens
the advocate wasn’t specifically interested in the case the health of wildlife. He’s merely making the
of children under three; rather, she was trying to counterpoint that organic farming also has a drawback
demonstrate that safety labels are more effective than as far as wildlife is concerned.
age-range labels in preventing toy-related injuries for (B) is similar to (A); the author doesn’t imply or assume
children of any age. anything about whether or not chemicals cause harm to
(D) suggests that safety labels wouldn’t be ver y wildlife. His only point is that organic farming cultivates
effective in reducing injuries, or at least injuries to a wider area, and thus reduces wildlife habitat.
those children whose parents don’t read labels. (C) The author is only interested in the effects that the
(E) focuses on a particular hazard, but the advocate two farming methods have on wildlife, so he needn’t
was speaking of toy-related injuries in general; she assume anything about their other disadvantages.
never said that safety labels would be especially (D) The author compared the acreage needed for total
effective in preventing choking. food production; he doesn’t get into what type of crops
are being produced, so he doesn’t really need to
11. (C) assume anything about that.
The whole stimulus is intended to prove the initial
statement that the toy-labeling law should require 13. (A)
explicit safety labels; the rest of the stimulus—the The stimulus is pretty simple: Reptiles have these
explanation of what toy-labeling laws currently require characteristics, therefore alligators—which we have to
and why the proposed change would be an know or at least accept as being reptiles—have ‘em,
improvement—is intended to support that statement. too. In the same way, (A) argues that whatever’s true
Clearly, the statement in question is the argument’s about green plants must be true about grass (which is
conclusion. a subset of green plants).
(A) As we’ve already said, the statement isn’t used to (B) “Some red butterflies” is more limited than the
support a further conclusion, the statement is the stimulus’ “[all] reptiles,” and one red butterfly is not a
conclusion. class or subset of a larger group, as alligators are of
(B) There are no “conflicting” goals discussed, and the reptiles or grass is of green plants.
statement isn’t a compromise; instead, it’s the single (C) would be parallel if it read, “All books provide
goal for which the advocate argues throughout the knowledge about the empirical world, so Woolf’s book
stimulus. does likewise.” But as (C) is written, that books can
(D) Huh?! Obviously, the advocate never attempts to provide such knowledge does not imply that each
refute her own conclusion! individual book does so.
(E) The statement isn’t a “particular instance,” it is (D) The Ali film is not a subset of Fassbinder films, it’s
itself the general principle under discussion; the only one particular example. To be parallel, (D) might have
particular instance discussed is the “three and up” read something like this: “All members of the Hanbury
label, and that’s an instance of age-range labeling. family have seen all of Fassbinder’s films, so Dierdre

21
PrepTest 15 Explained

Hanbury has seen all of his films.” Note that there the (A) Dr. Nash disagrees with Dr. Godfrey’s claim that
author would be assuming that Dierdre is a member of part-time work contributes to students’ academic
that family, just as our original assumes (or knows) that problems; she never denies that those problems are
alligators are reptiles. serious.
(E) features two separate terms—running a risk and (C) Dr. Nash accepts the accuracy of Dr. Godfrey’s
suffering a broken bone—in a way that the stimulus evidence—i.e., she accepts that there is a correlation
doesn’t. (E) would be closer if it read “Skiers run a between students working long hours part time and
high risk, so skier Lindsay runs a high risk.” doing poorly in school.
(D) Neither Dr. Nash nor Dr. Godfrey discusses the
14. (C) general question of whether schools are at fault for
The author is trying to establish that waste of taxpayer students’ academic problems; they discuss the much
dollars occurs at weapons-producing plants. As proof, more narrow problem of whether the school policy of
he cites the fact that the government has decided to allowing part-time work has contributed to students’
reopen a plant and allow it to violate environmental, academic problems.
health and safety laws, when it could just as easily (E) Dr. Nash believes that there is a relationship
produce the weapons at a safer facility. Didn’t between students’ academic problems and part-time
something get lost in the shuffle? What happened to employment, just not the relationship that Dr. Godfrey
taxpayer dollars? What does compliance with safety espouses.
laws have to do with wasting money? As (C) says, the
evidence doesn’t address the issue of wasting 16. (C)
taxpayer money, which is the subject of the argument’s
Since Dr. Godfrey says that students are having
conclusion.
academic problems as a result of working long hours,
(A) The main problem with the reasoning isn’t that the and Dr. Nash says students are working long hours
author hasn’t backed up the claim that the alternative because they have academic problems, it would be
site is safer, but that the issue of safety isn’t even the nice to see which came first: the poor academic
issue that the author set out to address. performance or the long work hours. As (C) says, if we
(B) The author’s point that weapons laboratories are could look at the students in question and see whether
money wasters doesn’t undermine his claim that they had academic problems before they started
production plants are also money wasters. working, we’d be in a better position to know whether
(D)’s distinction between “research” and “weapons” is the work could have caused their problems. (Obviously,
confusing; the research spoken of in the argument is if the problems came before work, the work didn’t
weapons research. The argument makes a distinction cause the problems).
between weapons research and weapons productions, (A) Neither Dr. Nash nor Dr. Godfrey said anything
and never confuses the two. about the connection between academic problems and
(E) The argument never compares research laboratories future career success, so this question is irrelevant.
to production plants except to say that they both waste (B) Whether or not students also spend long hours at
taxpayer money; they don’t have to be similar to be other activities doesn’t matter. Both disputants accept
compared in that respect. that there’s a connection between working long hours
and doing poorly in school; they only disagree about
15. (B) which condition causes the other, and (B) wouldn’t
Dr. Godfrey notices that the more students work at help clear that up.
their part time jobs, the worse they tend to do in (D) hints at some undescribed skullduggery, but doesn’t
school; he concludes that they’re doing badly because address the question of whether they’re getting poor
they’re working long hours. Dr. Nash also sees the grades because they’re working a lot or vice versa.
correlation between part-time work and poor grades, (E), like (A), speaks of what happens after high-school;
but she draws the opposite conclusion; she believes neither speaker says anything about this, so the
the students who work long hours do so because answer to the question posed in (E) wouldn’t help
they’re doing poorly in school. As (B) says, she offers resolve the dispute.
an alternative interpretation for the association noticed
by Godfrey.

22
Section III: Logical Reasoning

17. (A) (B) At best, (B) only mildly supports the idea that
X reasons that there is an “inevitable” trade-off bacteria mutations occur randomly; it doesn’t however
between human and animal welfare in medical support the conclusion that all genetic mutation is
research and that concern for human welfare should random.
prevail. Y responds by proposing an alternative way to (C) Since the stimulus doesn’t tell us that all genetic
conduct medical research. Y does not believe that the mutation in bacteria is random (it only mentions certain
only way to conduct medical research is on animals or kinds of bacteria), answer choice (C) doesn’t allow us
humans. Y suggests that research being done on to conclude anything. (C)’s conclusion would only be
animals could be done in some other way (e.g., true if all genetic mutations in bacteria were random.
computer modeling), without causing any suffering to (D) indicates that the genetic mutations of these
animals or to humans. Y rejects X’s claim that there particular kinds of bacteria would be random whatever
must be a “trade-off” between human and animal their environment, but doesn’t get us any closer to
welfare; or, as (A) says, Y contradicts a premise (that drawing the conclusion that all genetic mutation is
there must be an inevitable trade-off) on which X’s random.
argument relies.
(E) too concentrates on the bacteria. (E) suggests that
(B) Y does not disagree with X about the weight to be the experiment did a good job of duplicating the
given to animal suffering as opposed to human bacteria’s behavior in nature, at least in some
suffering. In fact, Y could even agree with X! If the only respects, but doesn’t allow us to draw conclusions
way to conduct medical research involved humans and about all genetic mutations.
animals, Y may agree that human interests are more
important than animal interests. We just don’t know 19. (A)
what Y believes about the relative weight of human and
According to the club rules, it’s necessary to be a
animal interests because Y does not compare the two.
member in good standing in order to vote. However,
All we know is that Y doesn’t believe that there is an
that doesn’t mean that being a member in good
inevitable trade-off; Y believes that the same results
standing automatically entitles one to vote (is
can be achieved by other means without animal
sufficient). Jeffrey is a member in good standing
suffering.
because he paid his dues, but remember that being a
(C) Y’s argument doesn’t present a logical consequence member in good standing isn’t sufficient to guarantee
of X’s premises; it contradicts X’s argument by denying he gets to vote. The club president could have
a premise on which X’s argument relies. disallowed his vote for another reason. Thomas makes
(D) Y disagrees with X’s argument; the new evidence the common mistake of confusing that which is
about other types of research is intended to refute X’s necessary with that which is sufficient. To use the
point. terminology in (A): Thomas fails to take into account
(E) Far from filling in a hole in X’s argument by supplying the distinction between something not being prohibited
an assumption, or unstated premise, Y attacks an (Jeffrey is not prohibited from voting because he is a
explicitly stated premise. club member in good standing and only club members
in good standing may vote) and its being authorized
18. (A) (every member in good standing automatically gets to
vote).
Question 18 involves a scope shift. The evidence is that,
in one particular environment, some types of bacteria (B) Thomas never attacks the president’s character
experienced random mutations. The conclusion is that but does focus on the question of Jeffrey’s eligibility to
all genetic mutation is random. The credited response vote. Thus, neither part of (B) is correct.
needs to link this specific example to the general (C) The only thing (C) could possibly be referring to is
conclusion. Combine the stimulus with (A). The stimulus Thomas’ statement in the second sentence “you’ve
states that some genetic mutations are random, and (A) admitted that club rules say…” But he’s not assuming
states that either all mutations are random or none are. that because Althea didn’t deny what the club rules say
If some mutations are random, then all mutations must that the club rules are true. He’s citing club rules as
be random; the alternative option isn’t possible. If (A) is independent evidence, and reminding his opponent
true, all genetic mutations are random, which is our that she’s already conceded this point.
conclusion.

23
PrepTest 15 Explained

(D) is irrelevant. The question is whether the President evidence, the “not always.” So what can we conclude
had the right to prevent Jeffrey, a member of good from the fact that EEGs can’t always detect the
standing, from voting. Thomas doesn’t need to discuss abnormal electrical impulses? How about (D): a
what they were voting about. negative reading—failure to pick up evidence of
(E) is irrelevant. Thomas’ argument is about Jeffrey’s abnormal impulses—doesn’t necessarily mean that
voting eligibility and doesn’t depend on whether Althea temporal lobe epilepsy is not present.
is an authority on club rules. Thomas’ argument (A) is absurd; a positive reading can’t “reasonably
doesn’t improve if she is an authority and isn’t weaker reliably” indicate two contradictory things.
if she isn’t. (B) The stimulus only discusses temporal lobe epilepsy,
so it would be inappropriate to introduce other forms of
20. (B) epilepsy in the conclusion.
People who consume calories beyond their proper (C) The stimulus never mentions erroneous positive
weight-maintaining level normally store fat and gain reading, only erroneous negative ones. Therefore, the
weight. But, people who regularly drink two or three correct conclusion can’t compare the relative accuracy
alcoholic beverage per day (a subset of all people) and of positive vs. negative readings.
exceed their weight-maintaining caloric intake usually
(E) distorts the case; although a negative reading is
don’t gain weight. To resolve the discrepancy, we want
not a sure indicator of the absence of temporal lobe
the choice that most explains why the general rule
epilepsy, in no sense is a negative reading an indicator
doesn’t apply to people who drink two to three
of the presence of such epilepsy.
alcoholic beverages a day. According to (B), when
people drink two or three alcoholic beverages a day,
22. (C)
the excess calories that they consume tend to be
“dissipated” as heat; so (B) explains why they don’t The quickest way to solve this Parallel Reasoning
convert those excess calories into fat and gain weight. problem is to consider the “flawed reasoning” directly.
The only way this conclusion must follow from the given
(A) says that some people who drink don’t exceed
evidence is if there’s roughly the same number of
their weight-maintaining caloric intake, but we want to
bicyclists in both age groups, and that’s a fact we can’t
know about those drinkers who do exceed their weight-
take for granted. If there are, for instance, 100
maintaining caloric intake.
bicyclists 18+ and 1,000 bicyclists under 18, then 90
(C) talks about people who do not drink alcoholic people with lights on their bikes would constitute a
beverages and therefore doesn’t explain anything majority in the 18+ category, whereas 100 people with
about people who do. Moreover, (C) doesn’t even lights on their bikes in the under 18 category would not
speak of people who consume calories beyond their constitute a majority of that group. In this way, we can
weight-maintaining limit; (C) speaks of people who eat take care of the evidence in the first sentence and still
“high-calorie foods,” a totally new idea. show how it doesn’t necessarily lead to the conclusion
(D) intensifies the myster y of the absence of weight in the second sentence. So the nature of the flaw is a
gain in alcohol-drinkers without explaining it. confusion of proportion with number. Scanning the
(E) opens up a whole new question (fewer calories) to choices, only (C) qualifies, with voters taking the role
ponder, but doesn’t enlighten us as to why the alcohol of bicyclists 18+, non-voters standing in for bicyclists
drinkers mentioned in the conclusion don’t put on the under 18, and “being on the mailing list” the equivalent
weight. of “having lights on their bikes.”
(A) In the original, the majority in one group possesses
21. (D) a trait while the majority of another group does not.
The stimulus tells us that EEGs can “often, but not (A), however, gives us the majority in the same group—
always” detect the abnormal electrical signals people in Sheldon—possessing different traits (buying
associated with temporal lobe epilepsy. We’re given gas on Monday and groceries on Tuesday).
the first part of the conclusion: In general, a positive (B)’s conclusion is a statement of causation (“the
EEG reading reliably indicates temporal lobe epilepsy. availability of videos was responsible”), which is
This corresponds nicely to the fact that EEGs can nothing like the stimulus.
“often” detect the signals associated with epilepsy. (D) can be rejected because its first piece of evidence
But as the “although” indicates, the rest of the is a statement about “every” member of a group, and
conclusion should correspond to the other part of the

24
Section III: Logical Reasoning

its conclusion evokes the concept of minimum (“must conclusions, both clearly marked by Keywords. The
have at least two fire trucks”), neither of which is stem in 24 alerts us to the relevant conclusion for this
present in the stimulus. Also, note that the concept one, namely: “Therefore,…the direction of the Earth’s
that is prevalent in both groups in the original, the magnetic field has changed over time.” The evidence
aspect of majority, is only present in part in (D). for this conclusion is presented in the first two
(E), like (D), veers away from the stimulus with a piece sentences: The direction of magnetization in solidified
of evidence about “everyone” in a particular group. lava reflects the direction of the Earth’s magnetic field
Also (E) involves two traits or behaviors, knitting and at the time the lava solidifies, and there are major
sewing, whereas the stimulus involves only one: having differences in this magnetization direction among
a light on one’s bike. solidified lava flows formed throughout the Earth’s
history. To show that this proves that the direction of
23. (E) the Earth’s magnetic field has changed over time, the
author must assume that once lava has solidified, the
The author’s conclusion is that the government should
direction of its magnetization is permanently fixed. If
not require removal of all asbestos insulation because
not—that is, if the direction of magnetization in
such removal disturbs asbestos, and when asbestos is
solidified lava can change unpredictably—then there’s
disturbed it becomes a health hazard. When left
no way that the differing magnetization directions
undisturbed, asbestos is harmless. (E) supports this
documented in the evidence supports the notion that
argument by suggesting a way in which asbestos
the Earth’s magnetic field has changed over time. The
removal may be worse for our health than leaving the
author must assume (C); otherwise, the differences
asbestos undisturbed. Not only does the initial removal
noted could be the result of unpredictable changes
pose a risk to our health, but, according to (E), the
within the solidified lava flows, not changes in the
removed asbestos remains a health hazard even after
Earth’s magnetic field at the time these lava flows
removal due to the risk that it will be disturbed again,
were formed.
resulting in the same negative consequences.
(A) The author doesn’t state that only lava can be
(A) The author is arguing that removing asbestos is
used to measure the Earth’s magnetic direction,
more dangerous than leaving it undisturbed; the fact
merely that it can be. Apply the Kaplan Denial Test: The
that there are other things more dangerous than
argument doesn’t fall apart if other things besides lava
asbestos is irrelevant to the argument and therefore
can reflect the Earth’s magnetic field direction as it
doesn’t support it.
existed in the past, which confirms that (A) is not a
(B) doesn’t strengthen the argument because 1) we necessary assumption.
already know that the removal of asbestos poses a
(B) fails to mention magnetization at all! Both the
health risk and 2) it implies that workers who wear their
evidence and conclusion deal with magnetization, so
“required” protective gear can avoid the health risk of
any key assumption must include this concept.
asbestos (which would weaken the argument).
Instead, (B) introduces the irrelevant concept of
(C) makes an irrelevant distinction between types of “consistencies.”
asbestos which doesn’t strengthen or weaken the
(D) includes an irrelevant comparison and, like (B),
author’s claim that it’s safer to leave asbestos alone
fails to mention magnetization. Apply the Kaplan Denial
(any kind of asbestos) than to remove it.
Test and it will become readily apparent that (D) is not
(D) actually weakens the argument by suggesting that a necessary assumption.
asbestos may be disturbed, and consequently become
(E) is outside the scope of the argument. Magnetized
a hazard, even if it is not deliberately removed; this
rocks? Hopefully, you dismissed this answer choice
weakens the author’s idea that it’s safe not to remove
quickly.
asbestos. If (D) were true, it wouldn’t be possible to
leave asbestos undisturbed; renovations or building
25. (D)
demolition would inevitably and unintentionally disturb
asbestos, thus posing a health risk even though the Now we’re asked to weaken the author’s second
asbestos wasn’t removed. conclusion (see the discussion for Question 24 above).
The author argues that lava flows that differ in age by
24. (C) several thousand years have roughly the same
magnetic direction, but that lava flows that differ in age
In a somewhat unusual argumentative structure (at
by much more time (hundreds of thousands or even
least by LSAT standards), the author draws two

25
PrepTest 15 Explained

millions of years) have different magnetic directions. result in a country being squeezed out of the global
Therefore, the author concludes, the Earth’s magnetic market. As for a consequence of the former (raising
field changes slowly. Any answer choice that weakens prices), we can’t infer a thing.
this argument must undermine the connection between (B) Adopting technology slower than competitors will
the slow passage of time and the slow change in lead to being squeezed from the global market, but the
direction. Choice (D), which states that the magnetic author never states that this is the only way a company
direction of lava can change rapidly (e.g., within two can be squeezed from the global market. There may be
weeks of a single flow’s solidification) undermines the other ways a competitor gets squeezed from the global
argument that the Ear th’s magnetic field, as market (e.g., political instability in a country) that don’t
represented by lava, changes slowly. involve technology. In other words, adopting technology
(A) doesn’t link the magnetic direction of lava to the slower than competitors is sufficient for being
Earth’s magnetic direction. It introduces the irrelevant squeezed out of the global market, but we can’t infer,
idea of “iron-containing liquids.” Besides, who cares as (B) does, that it’s a necessary condition.
how the magnetic direction changes? The issue is (C) Did you catch the scope shift in this one? The mere
whether it changes and how fast. Hopefully you dismissed adoption of new manufacturing techniques is not the
(A) quickly as outside the scope of the argument. issue; it’s whether a countr y’s manufacturers’
(B) introduces useless background information. What’s adoption of said techniques (or technology) is faster or
happened since scientists began measuring these slower than that of its foreign competitors. (C) fails to
magnetic directions in no way damages the link compare how quickly the different countries adopt
between the author’s evidence about the observation technology, and is thus not inferable.
of past lava flows and the conclusion drawn from it. (D) As seen above in (B), it’s possible for a group to
(C) is consistent with the idea that the change occurred be squeezed out of the market for a reason other than
slowly (several time in a few million years). Remember differences in the rate of technological adoption.
that we want to weaken the argument.
(E) fails to mention magnetic direction at all, and instead
focuses on an irrelevant aspect of the solidification
process.

26. (E)
The key here is to notice that each statement is versed
in formal logic. The tricky thing is that the testmakers
make use of equivalent phrases along the way. For
example, “fall more slowly” = “fall less rapidly.” It’s up
to you to make the logical connection when it comes to
different ways of saying the same thing. In short, it all
comes back to critical reading.
So here’s the argument: If X (slower to adopt new tech
than competitors), then Y (prod costs fall more slowly
than competitors). But if Y (prod costs fall less rapidly
= prod costs fall more slowly), then Z (prices fall less
rapidly). Choice (E) can be derived from using the
contrapositive on both of these statements: If NOT Z
(prices fall AS or more rapidly as competitors), then
NOT Y (prod costs fall AS or more rapidly). Continuing
back to the first statement, if NOT Y (prod costs fall AS
rapidly), then NOT X (not slower to adopt new tech = as
fast as competitors to adopt new tech). Put it
together, it adds up to (E).
(A) The concept of “raising prices” is never mentioned
in the original stimulus and is not the opposite of “not
lowering prices as rapidly.” The latter will definitely

26
Section IV: Logic Games

SECTION IV The Final Visualization: Here’s our master sketch and


rules:
LOGIC GAMES
HJKRST
Game 1: Student Speeches No HJK No J No J
No RST 1 2 3 4 5 6
Questions 1–6 H…S
No TJ or JT
The Action: Here’s a fairly standard sequencing game
in which we’re asked to order the speeches given by six The Big Picture:
students—H, J, K, R, S, and T. The Key Issues will
involve our typical sequencing concerns: • Some games simply don’t have any ear th-
shattering Key Deductions. While you should
1) Which speech is given in what time slot?
always attempt to combine the rules and seek
2) Which speeches can, must, or cannot be given out Key Deductions, you need to be able to
before what other speeches? recognize when a game is left fairly “wide open”
3) Which speeches can, must, or cannot be given after as is the case here. In such cases, don’t knock
what other speeches? your head against the wall—if you don’t see
anything, save valuable time by moving on to the
The Initial Setup: There are six speeches, so let’s write questions.
1 through 6 across the page. (You’d probably do exactly
• Even when there are no major Key Deductions,
that if, in real life, you had to keep track of the
don’t assume that there’s nothing of importance
students’ speeches.) Remember to list the students
to notice about the game before moving on to the
off to the side:
questions. Remember, not all entities are created
HJKRST equal. To identify the most important entity, look
for the one that appears in the most rules. Here,
1 2 3 4 5 6 J is in two of the rules, and the game tends to
The Rules: hinge on the placement of this entity. J can’t be
in 1 or 6 (Rule 4), and J is also prohibited from
1) This rule is tough to symbolize, but think it through being next to T (Rule 5). Some of the questions
and jot it down as best you can. We’re told that the are bound to be answered by T’s location limiting
speeches given by H, J, and K can’t all be given the possibilities for J’s placement. Whether
consecutively. “NO HJK” is fine for shorthand, as long directly or indirectly, the restrictions placed on J,
as you remember that these three entities can’t be including J’s relation to T, will lead you to the
consecutive in any order. For example, KHJ and JKH answer in quite a few of this game’s questions.
would violate this rule too.
• Think before you draw. Make sure you understand
2) Likewise, the speeches given by R, S, and T can’t be what a rule says before you even make a stab at
consecutive. “NO RST” will suffice as long as you realize symbolizing it. In order to successfully master
that this rule has the same implications as Rule 1. this game, you had to fully think through the
3) H’s speech is before S’s, so right away we know implications of Rules 1 and 2 before putting
that H can’t go 6th and S can’t go 1st. You could pencil to paper.
include these implications in your master sketch, but
they should be obvious enough that you need not even The Questions:
add them in. The important thing is to get the rule
down, so let’s shorthand it as “H…S.” 1. (D)
4) “NO J” over the 1 and 6 should do it for this rule. Use Kaplan’s technique for acceptability questions to
5) T’s speech can’t be given consecutively with J’s blow through this one in ten seconds. Compare each
speech. Let’s write “NO TJ or JT.” rule to each choice, and cross off any choice in
violation of any rule. Rule 2 is violated by (A). Rule 3
Key Deductions: See The Big Picture. lets us axe (E). Rule 4 eliminates (B), and Rule 5 does
away with (C) leaving us with (D).

27
PrepTest 15 Explained

2. (B) 6. (A)
With T in 3, J can’t go in 2 or 4 (Rule 5). And J can’t be After placing K in the 3 spot, there’s not much to do
in 1 or 6 (Rule 4), so the only space left for J is 5, but try out each choice. If H is in 4, we know that S
choice (B). must be in either 5 or 6. If S is in 5, the only spot open
(A), (C), (D), and (E) all could be true, but none must to J is 2. But if J is in 2, this gives us JKH which
be true. violates Rule 1. So S can’t be in 5. If S is in 6, the only
spot open to J is 5 (we just saw that J in 2 is no good
3. (C) in any case). But if J is in 5, we get KHJ consecutive,
which also violates Rule 1. (A) is impossible and is
With S and T in 3 and 4, respectively, Rule 5 prohibits
therefore our answer.
J from being next to T in 5. Of course, 1 and 6 are off
limits to J, so the only space open for J is 2. H has to (B) HSKJRT is an ordering that shows that this could
be before S (Rule 3), and the only open space before be true.
S is 1, so place H in 1. Since we can’t have STR in a (C) HJSKRT is an ordering that shows that this could
row (Rule 2), R can’t go in 5, so K must go in 5 and R be true.
must go in 6. The entire order is complete, and only (D) HTKSJR is an ordering that shows that this could
choice (C) corresponds. be true.
(A), (B), (D), and (E) are all impossible. (E) RJKTHS is an ordering that shows that this could
be true.
4. (A)
With K in 1 and H in 5, Rule 3 forces S into slot 6. From
there we’re left with J, R, and T to fill out spaces 2, 3
and 4. Rule 5 still forbids T and J from being next to
each other, so one of them has to be in 2 and the other
must be in space 4. R must separate J and T, so R
must be in 3, choice (A).
(B), (C), (D), and (E) all could be true, but need not be
depending on how you place T and J in the 2 and 4
slots.

5. (D)
This question stem creates the block SRK. Rule 3 adds
H to the mix (but not necessarily right before S) to give
us H…SRK. This block takes up four of our six spaces,
leaving only J and T. We’re plenty used to splitting up
these two by now thanks to Rule 5, so we can further
intuit that the H…SRK block can’t be in slots 1 through
4 or slots 3 through 6. Both of these situations would
force J and T next to each other. The block also can’t
take consecutive slots 2 through 5, because that
would leave J in either 1 or 6, which violates Rule 4. We
know then that either J or T must be in between H and
S with the other at one end of the sequence. But once
again we’re forced to remember that J can’t go at
either end, so J must come between H and S, leaving
T in either 1 or 6. So here are the only possibilities:
THJSRK or HJSRKT. The only statement that could be
true is choice (D).

28
Section IV: Logic Games

Game 2: Radar Areas The Rules:


The “rules” in the intro paragraph allowed us to
Questions 7–13 construct a picture of the physical layout of the radar
zones. The following indented rules tell us about the
At first glance, this game may look odd and rather radar zones in which the planes appear:
confusing, but don’t be intimidated simply because a
1) Our basic “loophole closer”: Each plane is in the
game looks scary. Keep your composure; the first step,
game and somewhere within the radar areas of our
as always, is to nail down the game’s action:
sketch.
The Action: Four circular radar areas—R, S, T, and U— 2) Pretty straightforward, but a little thought will let you
are within the country of Zendu (although the specific take it one step further: If J is in area S, then it’s either
country doesn’t matter, as you’ll see in the fifth bullet in the part of S that intersects T or it’s in the part of S
point under The Big Picture). Four planes—J, K, L, and that doesn’t intersect T. As long as you can visualize
M—are each in the air over Zendu in accordance with this, you can shorthand it any way you want. One
the rules. The rules are bound to clear the situation up possibility is to write a “J” in area S with one arrow
for us, but even at first glance you should be able to pointing to the space where S and T intersect and
intuit the Key Issues that the form the basis of the another arrow pointing to the rest of S that doesn’t
game: intersect T.
1) Which planes are in which radar areas? 3) “K ≠ J” should keep this rule nice and clear.
2) Which planes can, must, or cannot be in the same Whatever area one of these planes is in, the other
radar areas as which other planes? must be somewhere else. It would be difficult not to
automatically combine this with the previous rule to
The Initial Setup: An Initial Setup hinges mostly on the deduce right here and now that K is not in area S.
rules built into the opening paragraph. Part (not all) of 4) “L ≠ M” should do it for this one. Since we have no
area R intersects area T, so draw a circle for R and info so far on either of these planes, we can’t take this
another circle T that partially overlaps R. Next, we’re any further, so jot it down and move on.
told that part of area S intersects T. However, we can’t
5) M is in exactly one of the areas: M = 1 should suffice
add S to our sketch yet, because we don’t know about
to help us remember this.
S’s relationship to R; Ah, but now we do: The very next
clause in that sentence says that R and S don’t Key Deductions: Above, we combined Rules 2 and 3 to
intersect each other. So simply draw a circle for S that deduce that K cannot be in area S.
intersects T but not R. Finally, we’re told that area U is
Now let’s look more closely at Rule 5. Since M is in
completely within areas R and T. U must therefore fall
exactly one area, M cannot be in area U. Why?
in the space where R and T intersect, so draw another
Because If a plane is in area U, then it’s actually in
circle in this space and label it U. Be careful not to
three areas: U, R, and T. Can M be in area R? Sure, as
falsely assume that U takes up the entire area where
long as it’s not in the part of R that intersects with area
R and T intersect—this need not be the case. It’s quite
T. The same is true for M in areas T or S; it’s okay, as
possible for a plane to be in the area where R and T
long as M isn’t in the intersecting part.
intersect, yet still outside of area U. This is more work
than we’re used to for creating an Initial Setup, but in The Final Visualization: So, here’s our final, rather
this game, understanding and visually representing the circular, sketch:
Initial Setup is half the battle.

JKLM JKLM
K=J
L=M R
R U T
U T M=1 S
S K not in S J
M not in U

29
PrepTest 15 Explained

The Big Picture: The Questions:


• Force yourself to ask the relevant questions. 7. (D)
When you’re told that S and T intersect, you must
go further and ask “But wait, what about S and As with all acceptability questions, let’s just check the
R? How do they interrelate?” This is the kind of rules against the choices. Rule 1 kills (A) because K
active thinking that the Games section requires isn’t included. Rule 2 takes care of (B). Rule 3 knocks
and rewards. off (E). (C) is the only tricky one. Our Big Deduction
above allows us to cross it off because we know that
• Remember, critical reading isn’t a requirement M, which can only be in one area, can’t be in area U,
limited to the Logical Reasoning and Reading which is actually within two other areas. If this wasn’t
Comp sections—it’s vital in Logic Games as well, enough to kill (C), since area U is within area T, this
especially when interpreting rules. For example, choice places L and M in the same area (T), which
it’s extremely important that you recognize that violates Rule 4. In any case, the only choice left is (D),
just because area U is in the part of areas R and which must be the acceptable listing.
T that intersect, U doesn’t necessarily have to be
that entire area. Also, be sure that you didn’t get 8. (A)
the rule backwards and put all of area R inside of
area U. If K is in 2 of the areas, then K must be in the area
where R and T intersect (we already deduced that K
• Don’t be thrown just because a game is can’t be in S). In order for this to be possible, J must
unfamiliar. On some Games sections, the be in the part of S that doesn’t intersect T, since Rule
testmakers throw in a somewhat unfamiliar 3 forces K and J into different areas. (A) can’t be true.
game. While it won’t necessarily be the hardest
game on the section, they do want to see whether (B) and (C) must be true.
or not you can think on your feet, use your (D) and (E) could be true, but need not be.
common sense, and not be thrown for a loop just
because the setup is slightly different from a 9. (E)
more typical game. We just saw in the setup for question 8 that K could be
• If you’re having a hard time getting a handle on a in area T, which quickly kills (A), (B), and (C). The only
seemingly unfamiliar game, try out different difference between (D) and (E) is whether or not J can
situations. For example, try putting M in areas R, be in area T. And lo and behold, we see that J can be
S, T, and U and see what happens. Mentally work in T from the correct choice (D) to acceptability
out ways of separating L and M, or separating K question 7. Basically, there’s no reason why any of the
and J. This way you can quickly get a feel for planes cannot be in area T, and the answer is (E).
which planes can and can’t go where. Working out
a few “what if?” scenarios should make you feel 10. (E)
more comfortable with the game and help you get Here’s our Big Deduction staring us right in the face: M
a better handle on it. in area U? Can’t do it; M can only be in one area, but
• Don’t obsess over extraneous information. In this U is within two others. Choice (E) is the five-second
game, the fact that the zones fall within the answer. If you hadn’t made this deduction up front, or
country of Zendu doesn’t matter in the slightest. simply forgot about it, you could get the answer by
Did you draw a huge circle representing Zendu trying out the choices (which will no doubt take longer
and then try to put the radar areas inside? Why? than five seconds).
Did you worry about the planes being in the air at (A) is possible; we just saw from question 9 that
exactly noon? Why? The time frame never everyone could be in T.
changes; if it did, this would be more like a (B) K can be in U as long as J isn’t in T.
“process” game. Sometimes the testmakers
throw in this extra useless information in order to (C) is possible as long as M isn’t in R. You may also
get nervous test takers to waste as much time as have noticed that L was in R in one of the wrong (and
possible. Don’t fall for it. therefore possible) answer choices from question 8.
(D) is possible; once again we benefit from turning to
correct choice (D) for acceptability question 7 which
validly places M in R.

30
Section IV: Logic Games

11. (D)
A single plane can’t be in what two areas? We know
that S and R don’t intersect, so a plane couldn’t be in
both S and R. Not surprisingly, that’s too obvious, and
therefore not a choice. But we need go only one step
further: Since area U is completely within area R,
there’s no way for a plane to be simultaneously in
areas S and U. As easy as that, choice (D).
(A), (C) The areas listed in each of these choices
intersect, so it’s definitely possible for a plane to be in
both areas at once.
(B), (E) Area U is within areas R and T.

12. (E)
M is in area T. Since M is in exactly one area, so we
know that M is in the part of T that doesn’t intersect
with R or S. Rule 4 says that L and M can’t be in the
same area, so we know that L can’t be in the parts of
R or S that intersect with T. Thus L can’t be in U; L must
be in the non-intersecting part of area R or the non-
intersecting part of area S. At most, L can be in only
one area, which leads us to correct choice (E).
(A) through (D) are each quite possible.

13. (A)
The only way for a plane to be within three areas is to
be in area U, so L is in U (and therefore is also in R and
T). Rule 4 prohibits L and M from being in the same
area, so the only area open to M is S (which kills
choices (D) and (E)). We deduced way back that K
can’t be in area S, so axe (C). As we said before, the
only way to be in three areas is to be in area U. Since
S and U don’t intersect, J can’t be in three areas.
Cross off (B), which leaves us with our answer, (A).

31
PrepTest 15 Explained

Game 3: Car Pool Drivers Key Deductions: As with Game 1, no major deductions
leap off the page at us. However, J is in Rules 4 and 5,
Questions 14–19 so let’s take a closer look there. J must drive on
Wednesday, Saturday, or both (and possibly elsewhere,
The Action: Sequencing with one small twist: We’re remember). If the conditional in Rule 5 takes place
asked to schedule four people—Fritz, Gina, Helen, and (i.e., G drives on Monday), then J can’t be on Saturday
Jerry—on the days on which they drive, Monday and would have to drive on Wednesday (at least). Also
through Saturday. Here’s the minor twist—four drivers think about the “NO 2 IN A ROW” rule. If we have J on
for six days means that some folks have to drive on Wednesday, then we know that J can’t drive on Tuesday
more than one day. The Key Issues will be: or Thursday. Since this line of thinking is predicated on
1) Who drives on which day? a conditional, it won’t play out in every question.
Therefore, it isn’t absolutely necessary to think all this
2) Which people can, must, or cannot drive on
through in order to succeed in this game. However,
consecutive days?
noticing such things up front is always helpful in the
3) Which people drive before and after which other long run, and will help you to develop and to reinforce
drivers? the good habits and analytical thinking skills that will
be rewarded on test day.
The Initial Setup: Keep this simple; list the days
across the page and the drivers off to the side, like so: The Final Visualization: Here’s our final sketch:
FGHJ J at least
M T W TH F S No F
FGHJ M T W Th F S
No 2 in a row
The Rules:
If G on Mon, J not on Sat
1) This is a loop-hole closer, but unlike most loop-hole If J on Sat, G not on Mon
closers, it’s an important one. Every person drives at
least one day. This means that no driver is left out of
The Big Picture:
the schedule (which until this rule was a distinct
possibility). • Rule 4 is wordy and chock full of implications.
Often, a rule like this would mandate that Jerry
2) “NO 2 IN A ROW” should do it for this one. For
drive on Wednesday or Saturday, but not both.
example, if Gina drives on Monday, she can’t drive on
However, the testmakers threw us a slight curve
Tuesday.
here. No biggie—whenever you see a rule that’s
3) “NO F” over our M for Monday should remind us of slightly different from what you’re used to, take a
this. few seconds to think it through.
4) Break this rule up. “J” above the sketch with an • Often, the entity that appears in the most rules
arrow pointing to the “W” for Wednesday and the “S” will be the game’s most influential player. Never
for Saturday will take care of the first part of this rule. let this entity stray too far from your mind.
However, Jerry could drive on both Wednesday and
• If an entity is limited to one of two positions, it’s
Saturday as well as driving on other days. Add
often worth plotting out the results of each. We
something to your sketch to keep this added
call this investigating the “limited options.” But it’s
information in mind. We’ll write “at least” next to the J
also important to recognize when this strategy
in our sketch to remind us of this, but whatever way you
doesn’t apply: Here, we could look at what
came up with, including just memorizing the rule, is
happens if Jerry drives on Wednesday and then
fine as long as you come away with the correct
what happens when he drives on Saturday, but
interpretation.
since he can drive on both days, and on other days
5) “If G drives on Mon, J isn’t on Sat.” Remember to as well, discovering all of the possible options
jot down the contrapositive: “If J drives on Sat, G that result from Jerry’s situation would be a near
doesn’t drive on Mon.” impossibility. As stated in the Key Deductions
paragraph above, mentally manipulating a few
possible scenarios is probably all that’s necessary
before moving on to the questions.

32
Section IV: Logic Games

The Questions: 18. (B)


Nothing to do but check out the choices. If Gina drives
14. (D)
on Monday, Jerry can’t drive Saturday (Rule 5), which
This game too begins with an acceptability question. means that Jerry must drive on Wednesday. According
Pick a rule and compare. Rule 1 axes (E) since Fritz to Rule 2, he can’t drive on Tuesday as well. (B) can’t
isn’t included. Rule 2 does away with (A). Rule 4 kills be true and is the answer.
(C). Rule 5 takes care of (B), leaving us with (D).
(A) H F J F G J is fine, so (A) could be true.
15. (E) (C) and (E) G H J F J G shows that both of these could
be true.
Not much to do except try out the choices. We’ll get to
why the others are wrong, but first, if the order is G, F, (D) H J G F G J is acceptable, so (D) is not the answer.
J, H, F, G we see that (E) is possible and is our answer.
19. (E)
(A) This is a direct violation of Rule 4. We know that
Jerry drives on at least one of these days. Fritz drives twice, but the question stem and Rule 3 cut
his options down to Thursday, Friday, and Saturday. The
(B) If Gina drives on Monday and Wednesday, then
only way to get two days of driving out of him is for him
Jerry can’t drive on Saturday (Rule 5), which once again
to drive Thursday and Saturday (otherwise, he’d have
violates Rule 4.
to drive two days in a row, violating Rule 2). Since
(C) We know that Jerry drives on at least Wednesday Saturday is taken, Rule 4 forces Jerry to drive on
or Saturday, so if he also drives on Tuesday and Friday, Wednesday, at least. Jerry could also drive on Friday,
this would have him driving two days in a row. That’s a with Gina on Monday and Helen on Tuesday, so (E)
no-no according to Rule 2. could be true and is our answer.
(D) If Gina drives on Monday, then Rule 5 forces Jerry (A) Fritz can’t drive three times since the stem says
to drive on Wednesday (at least). If Jerry also drives on that he drives exactly twice. If any of the others drive
Thursday, he would be driving two days in a row, which three times, that wouldn’t leave enough days for
again violates Rule 2. everyone to take a turn driving.
(B) Fritz drives twice, so that leaves four days for the
16. (C)
other three people. No, three people can’t drive exactly
If Jerry drives on Wednesday and Saturday only, then once; one of them must drive twice to cover all six
the contrapositive of Rule 5 tells us that Gina can’t days.
drive on Monday. Rule 3 already axed Fritz from Monday,
(C) Jerry must drive on Wednesday which is immediately
and since in this one Jerry drives only on Wednesday
before Fritz’s day on Thursday.
and Saturday, Jerry can’t drive on Monday either. The
only person left for Monday is Helen, choice (C). (D) No, Jerry must drive on Wednesday.
(A), (B), (D), and (E) could be true, but need not be.

17. (A)
If Gina drives on Monday and Saturday only, then Jerry
must drive on Wednesday (at least) according to Rule
4. Fritz and Helen must each drive on at least one of
the remaining three days. This leaves one day open
that Fritz, Helen, or Jerry will drive on. Check the
choices, and we see that (A) must be true. Whoever
drives on that extra day will be the one other person
besides Gina to drive on exactly two days.
(B) No, the person who drives on Wednesday (i.e.
Jerry) could drive on Friday.
(C), (D) The possible ordering G H J F J G kills both of
these choices.
(E) No, Fritz could be the driver on Tuesday (e.g. G F J
H J G).

33
PrepTest 15 Explained

Game 4: Plumber Teams possible for Frank to team up with another experienced
plumber. If you like, you can write “If F, then Fs or F +
Questions 20–24 X (experienced).”

The Action: This game involves distributing plumbers The Final Visualization: Here’s what we’re armed with
into four groups of two, which suggests a grouping heading into the questions:
distribution action. Technically, there’s a selection
EX inex. No FM
element as well: We need to select eight of the nine
plumbers. However, it’s probably easier to think in
FGJKM rstn No Fr
terms of having to leave one plumber out as opposed No Fn
to having to choose eight of the nine. The plumbers are If t, then G or K
chosen from five experienced plumbers—F, G, J, K, and X X X X No Jr
M—and four inexperienced plumbers—r, s, t, and v. No Ft
Let’s use capital letters for the experienced plumbers If F, then Fs or F + X
and lowercase for the inexperienced ones. The Key
Issues will be: The Big Picture:
1) Which two plumbers are assigned to each team? • Always work out a game’s number situation in
advance, by asking yourself the relevant
2) Which plumbers can, must, or cannot be assigned
questions. In this case: “How can I split five
to the same team?
experienced and four inexperienced plumbers into
The Initial Setup: We have four teams of two, so let’s four groups of two?” Answer: Always leave one
put down four sets of two dashes. List the entities off out. Then, when you get more information,
to the side: expand on what you know. As a consequence of
Rule 2, you should ask yourself: “How do the
EX inex. numbers work now?” Answer: We must include
FGJKM rstn four experienced and four inexperienced
plumbers OR five experienced plumbers (giving us
one team made up only of experienced plumbers)
and three inexperienced ones.
The Rules: • In three of the four games on this section it’s
1) Basically a loop-hole closer, but the game would be beneficial to identify the most important entity,
undoable without it. which is normally the entity that appears in the
most rules. Here it’s obviously Frank. Once again,
2) This rule narrows down the possible pairings there are no earth-shattering deductions to be
enormously. Let’s write an “X” for “experienced” under made in this game, but thinking through Frank’s
one of the dashes in each of the four teams. situation in advance greatly helps in answering
3) Three rules for the price of one: “NO FM”—“NO Fr” the questions.
—“NO Fv.” These three restrictions tell us quite a bit • Always turn negatives into positives. Knowing
about Frank’s situation. who doesn’t pair up with Frank is not nearly as
4) “If t, then G or K” tells us that only these two valuable as knowing who does or can (in this
experienced plumbers may team up with the case, Sally or one of the “X” plumbers).
inexperienced Tim.
5) One rule for the price of one: “NO Jr” will do it. The Questions:
Key Deductions: Rule 3 severely limits Frank’s options, 20. (C)
and Rule 4 limits it further. If Tim may only be teamed Here’s our deduction about Frank. The only
with Gene or Kathy, then obviously Tim may not team inexperienced plumber that Frank can team up with is
up with Frank. Add “NO Ft” to your list of Frank rules. Sally, choice (C). If you don’t remember how we
This means that Sally is the only inexperienced deduced this, look back at our work under Key
plumber that Frank may team up with. Does that Deductions. (Note that Kathy, choice (A), is not an
mean “always Fs?” No; remember that we have five inexperienced plumber.)
experienced plumbers and need only four of them. It is

34
Section IV: Logic Games

21. (C)
This is similar to an acceptability question. Rule 3
allows us to cross off (A) and (B). Rule 2 says that
each team must have at least one experienced
plumber, so there goes (D) and (E). We’re left with (C).

22. (B)
Take it one step at a time: Tim is assigned to a team,
and Sally is assigned to another. Sally’s partner is a
plumber who could have been Tim’s partner. Tim can
only partner with Gene and Kathy. So of G and K, one
of them will go with Tim and the other will go with Sally.
Since Sally isn’t teamed with Frank, then Frank can
only be teamed with another experienced plumber
(besides Gene and Kathy who are already taken). Mark
is no good thanks to Rule 3, which leaves only Jill,
choice (B).

23. (A)
If Gene is out, we know that 1) all of the other plumbers
are used, and 2) Tim’s partner is Kathy (see Rule 4).
Since exactly four experienced plumbers are used, we
must split them all up. Frank must therefore be teamed
with an inexperienced one, which means Sally. Rule 5
says that Jill and Roberta can’t be teamed up, so Jill
must be teamed with the only inexperienced plumber
left, Vernon choice (A).

24. (A)
All of the inexperienced plumbers are used, so one of
our experienced plumbers is the one to be left out. R,
T, and V aren’t with Gene, so either Gene is teamed
with Sally or Gene is the one left out. If Gene is teamed
with Sally, then Frank’s only possible inexperienced
partner is taken, which means that he’s the one left
out. Sally must therefore be teamed with either Frank
or Gene, choice (A).

35
1-800-KAP-TEST | kaptest.com

ÖLL3113Afä
LL3113A

*LSAT is a registered trademark of the Law School Admission Council. Printed in USA ©2008 Kaplan, Inc.

You might also like